Входная контрольная работа обществознание 10 класс: Входной тест по обществознанию 11 класс

Содержание

Входные тестовые задания. обществознание. 10 класс.

Обществознание. Входные тестовые задания. 10 класс Вариант №1

1.Гражданин К.был уволен с работы. Для установления законности увольнения он может обратиться 1) к депутату парламента 2) в суд 3) к президенту 4 )в профсоюз 2.В Государственной Думе происходит обсуждение нового законопроекта о льготном налогообложении Какой вид деятельности иллюстрируется этим примером? 1) политическая 2)экономическая 3)духовная 4)социальная 3. Человек в отличие от животного способен 1)совершать привычные действия 2)предварительно обдумывать свои действия 3) проявлять эмоции 4)заботиться о потомстве 4. Страна А.с населением в 15млн.человек расположена в южном полушарии. Какая дополнительная информация позволит судить о принадлежности страны А. к обществам традиционного типа? 1)верховная власть в стране передается по наследству 2)слабо развита сеть услуг 3)в стране проживает многонациональное население 4)основу хозяйства страны составляет аграрное производство

5. В государстве М. в руках президента сосредоточены полномочия главы правительства и главы государства. Какая форма правления в государстве М. а) президентская республика б) парламентская республика в) федеративное государство г) унитарное государство 6. Верны ли следующие суждения о демократическом государстве? А. В демократическом государстве исключены случаи нарушения прав человека Б. В демократическом государстве закон гарантирует защиту прав национальных меньшинств а) Верно только А. б) Верно только Б в) Оба суждения верны г) Оба суждения неверны 7.
Анна Петровна активный и творческий человек. Она выступила инициатором проведения субботника в своём дворе. Это характеризует её как:

1)личность;                                                     3) интеллигентного человека;

  1. администратора;                                        4) профессионала;

8. Верны ли следующие суждения о сферах общественной жизни? А.  Экономическая сфера регулирует организацию государственной власти; Б.  Экономическая сфера обеспечивает удовлетворение материальных потребностей общества;

1) верно только А;                                                       3) верны оба суждения; 2) верно только Б;                                                       4) оба суждения неверны;

9. В стране В. активно развиваются наукоёмкие производства, произошли революционные изменения в сфере массовых коммуникаций. На производстве и в быту применяются компьютеры, робототехника.  Уровень образованности населения очень высок. К какому типу относится общество Б.?

  1. традиционному;                                             3) аграрному;

  2. индустриальному;                                   4) информационному

10. Верны ли следующие суждения о глобальных проблемах? А. Глобальные проблемы угрожают существованию человечества как биологического вида;  Б. Глобальные проблемы не могут быть решены в отдельных странах, регионах мира; 1) верно только А;                                                             3) верны оба суждения; 2) верно только Б;                                                             4) оба суждения верны 11. Социальной потребностью человека является потребность в:

  1. пище и воде;                                                        3) физическом движении;

  2. воздухе;                                                                 4) общении;

12. Какие отношения связаны преимущественно с политической сферой жизни общества? 1)производителей и потребителей;                 2) партий и государства;     3) родителей и детей                            4) писателей и читателей; 13. Какой пример иллюстрирует связь общества и природы?

               1) создание экологических заказников и заповедников;

               2) проведение реформы системы образования;

               3) существование сословного деления общества;

               4) очередные выборы депутатов парламента; 14. Верны ли следующие суждения личности?

  А. Личность проявляется в физических качествах и особенностях человека   Б. Формирование личности происходит на протяжении всей жизни человека. 1) верно только А;                  2) верно только Б;                            

 3) верны оба суждения;    4) оба суждения неверны;

15. . Установите соответствие между правомочиями собственника и их содержанием

Правомочия Содержание

а) владение 1) возможность эксплуатации

имущества путем извлечения из

него полезных свойств

б) пользование 2) нахождение вещи в

собственности

в) распоряжение 3) возможность определения

юридической судьбы вещи

путем изменения ее

принадлежности, состояния.

16. Передача или продажа государственного имущества в частную собственность

а)приватизация б) национализация в) протекционизм г) меркантилизм

17. Нормы права в отличие от норм морали 1)регулируют общественные отношения 2) обеспечиваются силой общественного мнения 3) соответствуют общепринятым представлениям о добре и зле 4) выражаются в официальной форме

18. Конституционной обязанностью гражданина является

1) работа на предприятии 2) участие в выборах 3)защита Отечества 4) членство в политических партиях 19. Верны ли следующие суждения о возникновении правоспособности и дееспособности. А. Правоспособность физического лица наступает с момента получения паспорта Б. Полная дееспособность физического лица наступает с 18 лет. 1) верно только А;                           2) верно только   Б;                      3) верны оба суждения;                                                             4) оба суждения неверны;

20. Здания заводских цехов, производственные помещения, станки относятся к одному из основных факторов

1) капиталу

2)земле

3)предпринимательству

4)труду 21. В приведённом списке указаны черты сходства и различия традиционного и индустриального обществ.  Выберите и запишите в первую колонку таблицы порядковые номера черт сходства, а во вторую – порядковые номера черт отличия.  

    1. развитие промышленности; 2) наличие норм общественной жизни   3) признание прав и свобод граждан; 4) создание культурных ценностей;

Черты сходства

Черты отличия

22. Установите соответствие

Факты                                                                                      Сферы жизни общества

А) производство товаров и услуг;                                     1) экономическая;

Б) отношения «отцов» и «детей»;                                    2) социальная;

В) межнациональный конфликт;

Г) оказание банковских услуг;

23. Прочитайте приведенный текст, каждое положение которого обозначено соответствующей буквой.

А.Проблема сохранения мира является одной из глобальных проблем современности.

Б.Считаем, что обращение к мировому сообществу с призывом к разоружению крайне необходимо. В. Проведенные расчеты показывают, что существующий арсенал оружия способен уничтожить все живое на земле.

Определите, какие положения текста имеют

  1. фактический характер

  2. оценочный характер

Запишите в таблицу

24. Ниже приведен ряд терминов. Все они, за исключением двух, характеризуют понятие «факторы производства». Найдите два термина, «выпадающих» из общего ряда, и запишите цифры, под которыми они указаны.

1) Налоги, 2) предпринимательские способности, 3) капитал, 4) земля, 5) труд, 6) акции.

25.. Владелец фирмы решил подарить уходящему на пенсию сотруднику автомобиль. Этот пример иллюстрирует право собственника

а) распоряжаться имуществом

б) пользоваться имуществом

в) владеть имуществом

г) наследовать имущество

Вариант 2.

1.Установите соответствие между правомочиями собственника и их содержанием

Правомочия Содержание

а) владение 1) возможность эксплуатации

имущества путем извлечения из

него полезных свойств

б) пользование 2) нахождение вещи в

собственности

в) распоряжение 3) возможность определения

юридической судьбы вещи

путем изменения ее

принадлежности, состояния.

  1. Кто является источником власти в демократическом обществе?

а) народ

б) представители крупного бизнеса

в) передовой общественный класс

г) интеллектуальная элита общества

  1. Верны ли следующие суждения о политике? Политика – это деятельность

А. по управлению государством

Б. по осуществлению власти в обществе.. 1) верно толькоА;                                         2) верно только Б;    3) верны оба суждения;     4) оба суждения неверны;

4. Полезные ископаемые, вода, лес относятся к одному из основных факторов

1) капиталу

2)земле

3)предпринимательству

4)труду

5. Верны ли следующие суждения о возникновении правоспособности и дееспособности. А. Правоспособность физического лица наступает с момента получения паспорта Б. Полная дееспособность физического лица наступает с 18 лет.   1) верно только А;                                              3) верны оба суждения 2) верно только Б;                                                          4) оба суждения неверны

А6. Гражданин К.был уволен с работы. Для установления законности увольнения он может обратиться 1) к депутату парламента 2) в суд 3) к президенту 4 )в профсоюз А7. В Государственной Думе происходит обсуждение нового законопроекта о льготном налогообложении Какой вид деятельности иллюстрируется этим примером? 1) политическая 2)экономическая 3)духовная 4)социальная А8. Человек в отличие от животного способен 1)совершать привычные действия 2)предварительно обдумывать свои действия 3) проявлять эмоции 4)заботиться о потомстве А9. Страна А.с населением в 15млн.человек расположена в южном полушарии. Какая дополнительная информация позволит судить о принадлежности страны А. к обществам традиционного типа? 1)верховная власть в стране передается по наследству 2)слабо развита сеть услуг 3)в стране проживает многонациональное население 4)основу хозяйства страны составляет аграрное производство А10. В государстве М. в руках президента сосредоточены полномочия главы правительства и главы государства. Какая форма правления в государстве М. а) президентская республика б) парламентская республика в) федеративное государство г) унитарное государство А11. Верны ли следующие суждения о демократическом государстве? А. В демократическом государстве исключены случаи нарушения прав человека Б. В демократическом государстве закон гарантирует защиту прав национальных меньшинств а) Верно только А. б) Верно только Б в) Оба суждения верны г) Оба суждения неверны А12. Верны ли следующие суждения о налогах А. Налоги – единственный источник финансирования государственных программ. Б. Налоги – обязательные платежи граждан и хозяйственных организаций , взимаемые в пользу государства. 1) Верно только А 2) Верно только Б 3) Верны оба суждения 4) Оба суждения неверны

А13. Верны ли следующие суждения о сферах общественной жизни?

А.  Экономическая сфера регулирует организацию государственной власти;

Б.  Экономическая сфера обеспечивает удовлетворение материальных потребностей общества;

1) верно только А;                                                       3) верны оба суждения; 2) верно только Б;                                                       4) оба суждения неверны;

А14. В стране В. активно развиваются наукоёмкие производства, произошли революционные изменения в сфере массовых коммуникаций. На производстве и в быту применяются компьютеры, робототехника.  Уровень образованности населения очень высок. К какому типу относится общество Б.?

1)традиционному;                                             3) аграрному;

2)индустриальному;                                   4) информационному

А15. В приведённом списке указаны черты сходства и различия традиционного и индустриального обществ.    Сначала выпишите черты сходства, а потом различия. 1) развитие промышленности;   2) наличие норм общественной жизни   3) признание прав и свобод граждан; 4) создание культурных ценностей;

А16.. Социальной потребностью человека является потребность в:

  1. пище и воде;                                                        3) физическом движении;

  2. воздухе;                                                                 4) общении;

А17. Верны ли следующие суждения о личности?

                А. Личность проявляется в физических качествах и особенностях человека   Б. Формирование личности происходит на протяжении всей жизни человека.   1) верно только А;                   2) верно только Б;                                                          3) верны оба суждения;   4) оба суждения неверны;

А18. Получение наследства регулируется правом

а) административным

б) семейным

в) трудовым

г) гражданским 19.. Признаком тоталитарного режима является: а) монополия на власть одной политической партии б) невмешательство государства в дела гражданского общества в) обязанность граждан подчиняться законам г) осуществлении выборов в органы государственной власти на альтернативной основе

20. Нормы права в отличие от норм морали а) регулируют общественные отношения б) обеспечиваются силой общественного мнения в) соответствуют общепринятым представлениям о добре и зле г) устанавливаются государством

21. Переход предприятий из рук частных предпринимателей в государственную собственность составляет суть

а) национализации б) инфляции

в) приватизации г) разгосударствления

22. Продажа или безвозмездная передача государственного имущества в собственность физических и юридических лиц называется _____________

23.Верховенство и полнота власти внутри страны и ее независимость во внешней политике – это

а) политический режим

б) форма правления

в) государственный суверенитет

г) форма территориального устройства

24. Верны ли следующие суждения о налогах

А. Налоги – единственный источник финансирования государственных программ.

Б. Налоги – обязательные платежи граждан и хозяйственных организаций , взимаемые в пользу государства.

1) Верно только А

2) Верно только Б

3) Верны оба суждения

4) Оба суждения неверны

25. Хозяйственная деятельность, осуществляемая на свой страх и риск , направленная на получение прибыли называется _____________.

Входные тестовые задания 11 класс Обществознание.

Вариант I.

  1. Запишите пропущенное слово.

Элементы социального контроля.

Элементы социального контроля

Характеристики

Социальные нормы

Система запретов, ограничений, разрешений, основанныхна поддержке большинства членов общества

Социальные…………

Общественная реакция на поведение человека или группы

  1. Найдите понятие, которое является обобщающим для всех остальных понятий и запишите цифру, под которой оно указано

    1. Этнос 2) профессиональная группа 3)малая группа 4)большая группа 5)социальная группа

3.Ниже приведен ряд терминов. Все они, за исключением двух, представляют виды искусства Найдите два термина и запишите их номера.

1)театр 2)библиотека 3)музей 4)кино 5)музыка 6)живопись

4. Выберите верные суждения о человеке и запишите цифры, под которыми они указаны

1) Духовные потребности человека связаны с его биологической сущностью

2) Индивид – единичный представитель человечества

3) К социальным потребностям относят потребности в пище, воде. Воздухе

4) свобода человека включает в себя возможность выбора между добром и злом

5) Личность — совокупность социально значимых качеств индивида, формирующихся в процессе общественной жизни.

5. Установите соответствие между формами и этапами познания

ФОРМЫ ПОЗНАНИЯ ЭТАПЫ ПОЗНАНИЯ

А) ощущение 1) чувственное познание

Б) понятие 2) рациональное познание

В) восприятие

Г) представление

Д) суждение

6. Критик отозвался о новом художественном фильме как о произведении элитарной культуры. Запишите цифры, под которыми указаны признаки элитарной культуры

1) ориентация на получение коммерческой выгоды

2) учет запросов самых широких слоев населения

3) сложность форм художественной выразительности

4) развлекательный характер

5) необходимость специальной подготовки для понимания смысла произведения

6) ярко выраженная авторская позиция

7. В стране Zразвито фабричное производство. Какие иные признаки свидетельствуют о том, что она развивается как общество индустриального типа.

1) Происходит демократизация политической жизни. Провозглашены политические свободы

2) Религия оказывает значительное влияние на развитие общества

3) Формируется класс промышленных рабочих

4) Существует разделение труда

5) Происходит механизация производства

6) Развивается сельское хозяйство

8. В стране X активно развивается крупное машинное производство, растут города. Работники добились от правительства принятия справедливого законодательства о труде. Какие признаки подтверждают наличие в стране рыночной экономики?

1) В стране каждый собственник факторов производства свободно распоряжается ими

2) С/х производит широкий ассортимент продукции на экспорт

3) собственниками земли, предприятий являются частные лица

4) Промышленное производство растет быстрыми темпами

5) валюта страны высоко ценится на международном рынке

6) В стране существует конкуренция производителей

9.Установите соответствие

Характеристики Факторы производства

А) все виды ресурсов, имеющихся на планете и пригодных 1) труд для производства экономических благ 2) земля

Б) деятельность людей по производству товаров и услуг 3) капитал путем использования их физических и интеллектуальных возможностей, навыков и опыта

В) факторный доход – заработная плата

Г) здания, сооружения, оборудование

Д) объем денежной массы

10. Выберите верные суждения о социальной мобильности

1) К горизонтальному виду мобильности относится получение офицером внеочередного воинского звания

2) К вертикальному виду мобильности относится переход человека в более низкий социальный статус

3) Социальная мобильность – это разделение общества на группы, занимающие разное социальное положение

4)Межпоколенная мобильность — -сравнительное изменение социального статуса у разных поколений.

11. В стране К правительство формируется блоком политических партий, победившим на парламентских выборах. Найдите в списке черты, свидетельствующие, что в стране К выборы парламента проходят по пропорциональной системе

1) Голосование проводится по спискам политических партий

2) Существует возможность выдвижения независимых беспартийных кандидатов

3) Граждане голосуют прежде всего за программы партий, а не за конкретных людей.

4) Количество мест, полученных партией в парламенте зависит от процента голосов, поданных за партию на выборах.

5) Победу одерживает кандидат, получивший большинство голосов на выборах

6) Предусмотрено голосование по одномандатным округам.

12. Что из перечисленного относится к конституционным обязанностям гражданина РФ?

1) сохранение исторического и культурного наследия

2) уплата законно установленных налогов и сборов

3) защита Отечества

4) участие в управлении делами государства

5) выбор профессии

13. Установите соответствие

СУБЪЕКТЫ УЧАСТНИКИ УГОЛОВНОГО СУДОПРОИЗВОДСТВА

А) прокурор 1) сторона защиты

Б) следователь 2) сторона обвинения

В) обвиняемый

Г) потерпевший

Д) защитник

14. Выберите верные суждения об отклоняющемся поведении

1) отклоняющееся поведение индивида всегда связано с нарушением нравственных норм 2) отклоняющееся поведение индивида может быть вызвано его социальным окружением 3) отклоняющееся поведение индивида может иметь как негативный характер, так и позитивный 4) отклоняющееся поведение проявляется только у индивидов и не свойственно социальным группам 5) отклоняющееся поведение влечет определенные социальные санкции

15. Государство Н включает в себя несколько субъектов, обладающих определенной самостоятельностью; регулярно проходят всеобщие равные прямые, на основе тайного голосования выборы главы государства и парламента. В стране представлен широкий спектр политических идеологий, права и свободы граждан гарантированы законом. Найдите в списке характеристики формы государства

1) монархия

2) республика

3) федеративное государство

4) демократическое государство

5) тоталитарное государство

6) унитарное государство

16. Установите соответствие между действиями и элементами правового статуса работника в РФ

ДЕЙСТВИЯ ЭЛЕМЕНТЫ ПРАВОВОГО СТАТУСА

А) соблюдать трудовую дисциплину 1) права

Б) получать полную информацию об условиях труда 2) обязанности

В) выполнять установленные нормы труда

Г) своевременно и в полном объеме получать зарплату

Д) бережно относиться к имуществу работодателя

17. Анна Петровна активный и творческий человек. Она выступила инициатором проведения субботника в своём дворе. Это характеризует её как:

1)личность;                                                     2) интеллигентного человека;

  1. администратора;                                        4) профессионала;

18. Установите соответствие между отличительными признаками и типами обществ.

ОТЛИЧИТЕЛЬНЫЕ ПРИЗНАКИ ТИПЫ ОБЩЕСТВ

А) основополагающая роль религии 1) аграрное

Б) сословная структура общества 2) индустриальное

В) серийное производство товаров массового потребления 3) постиндустриальное

Г) выдвижение на первый план сферы услуг

Д) ведущая роль информации в жизни общества

19. На графике отражено изменение предложения моркови и лука на рынке: кривая предложения переместилась из положения S в положение S1. Какие из перечисленных факторов могут вызвать такое изменение? 1) увеличение доходов производителей моркови и лука 2) гибель урожая в результате неблагоприятных погодных условий 3) уменьшение платы за аренду земельных участков 4) снижение цен на дизельное топливо и бензин 5) повышение цен на посадочные материалы

20. А. был избран губернатором области. Он уделяет большое внимание защите прав и свобод граждан, развитию институтов гражданского общества. В общении он доброжелателен. Открыт для критики. Что из перечисленного характеризует тип политического лидерства. Запишите цифры, под которыми указаны верные характеристики

1) общенациональный 2) демократический 3) традиционный 4) авторитарный 5) региональный 6) харизматический

21. Вставьте необходимые слова на место пропусков.

Социальные нормы — ___________(А), регулирующие взаимоотношения людей и их объединений. Общеобязательные, формально-определенные правила поведения, которые установлены или санкционированы, а также охраняются государством, называются __________(Б). Сложившиеся в обществе правила поведения, которые выражают представления людей о добре и зле, справедливости и несправедливости, долге называются __________(В). Действие этих норм обеспечивается внутренним убеждением, ____________-(Г), мерами общественного воздействия. В результате многократного повторения, в силу привычки исполняются _____(Д). есть и иные виды социальных норм, и все они выполняют роль регулятора __________(Е).

Список терминов:

1)общественное мнение 2) общественное отношение

3) социальные группы

4) корпоративные нормы

5) социальная динамика

6) нормы обычаев

7) правила поведения

8) нормы морали

9) нормы права

Входные тестовые задания 11 класс Обществознание.

Вариант II.

  1. Запишите пропущенное слово.

Типы избирательных систем.

Типы избирательных систем

Характеристики

………………..

Система формирования выборных органов через персональное представительство. Избранным считается кандидат. Набравший большинство голосов избирателей

Пропорциональная

Система формирования выборных органов власти через партийное представительство

  1. Найдите понятие, которое является обобщающим для всех остальных понятий и запишите цифру, под которой оно указано

    1. Социальный контроль 2) мораль 3)право 4)поощрение 5)наказание

3. Ниже приведен ряд терминов. Все они, за исключением двух, характеризуют социальную динамику. Найдите два термина и запишите их номера.

1)прогресс 2)структура 3) эволюция 4)реформа 5) революция 6)система

4. Выберите верные суждения о человеке и запишите цифры, под которыми они указаны

1) Духовные потребности человека связаны с его биологической сущностью

2) Индивид – единичный представитель человечества

3) К социальным потребностям относят потребности в пище, воде. Воздухе

4) свобода человека включает в себя возможность выбора между добром и злом

5) Личность — совокупность социально значимых качеств индивида, формирующихся в процессе общественной жизни.

5. Установите соответствие между формами и этапами познания

ФОРМЫ ПОЗНАНИЯ ЭТАПЫ ПОЗНАНИЯ

А) ощущение 1) чувственное познание

Б) понятие 2) рациональное познание

В) восприятие

Г) представление

Д) суждение

6.. В художественном произведении присутствует национальный колорит. Какие признаки позволят сделать вывод о том, что произведение относится к массовой культуре? Запишите цифры, под которыми они указаны.

1) у этого произведения неизвестный автор

2) оно передается от поколения к поколению

3) его создатели получили хорошую прибыль

4) его исполняют по радио профессиональные актеры

5) оно носит развлекательный характер

6) оно имеет сложное содержание

7. В стране К. большинство населения проживает в городах-мегаполисах. Найдите в списке признаки , позволяющие сделать вывод о том, что страна развивается как индустриальное общество.

1) в стране преобладают наукоемкие производства

2)в стране происходит механизация производства

3) в стране сословная социальная структура

4) в экономике страны преобладает массовое промышленное производство

5) в стране граждане добились равноправия и избирательных прав

6) в стране активно развивается виртуальная культура

8. В стране , расположенной на морском побережье, активно развивается туристическая отрасль. Какие признаки подтверждают наличие в стране рыночной экономики?

1) В стране каждый собственник факторов производства свободно распоряжается ими

2) В стране производится широкий ассортимент товаров народного потребления

3) Собственниками земли, предприятий являются частные лица

4) Правительство вкладывает значительные средства в строительство дорог

5) В столице государства открыт новый международный аэропорт

6) В стране существует конкуренция производителей

9.Установите соответствие

Характеристики Факторы производства

А) все виды ресурсов, имеющихся на планете и пригодных 1) труд для производства экономических благ 2) земля

Б) деятельность людей по производству товаров и услуг 3) капитал путем использования их физических и интеллектуальных возможностей, навыков и опыта

В) факторный доход – заработная плата

Г) здания, сооружения, оборудование

Д) объем денежной массы

10. Выберите верные суждения о социальной мобильности

1) К горизонтальному виду мобильности относится получение офицером внеочередного воинского звания

2) К вертикальному виду мобильности относится переход человека в более низкий социальный статус

3) Социальная мобильность – это разделение общества на группы, занимающие разное социальное положение

4)Межпоколенная мобильность — -сравнительное изменение социального статуса у разных поколений.

11. В стране К правительство формируется блоком политических партий, победившим на парламентских выборах. Найдите в списке черты, свидетельствующие, что в стране К выборы парламента проходят по пропорциональной системе

1) Голосование проводится по спискам политических партий

2) Существует возможность выдвижения независимых беспартийных кандидатов

3) Граждане голосуют прежде всего за программы партий, а не за конкретных людей.

4) Количество мест, полученных партией в парламенте зависит от процента голосов, поданных за партию на выборах.

5) Победу одерживает кандидат, получивший большинство голосов на выборах

6) Предусмотрено голосование по одномандатным округам.

12. Что из перечисленного относится к конституционным обязанностям гражданина РФ?

1) сохранение исторического и культурного наследия

2) уплата законно установленных налогов и сборов

3) защита Отечества

4) участие в управлении делами государства

5) выбор профессии

13. Установите соответствие

СУБЪЕКТЫ УЧАСТНИКИ УГОЛОВНОГО СУДОПРОИЗВОДСТВА

А) прокурор 1) сторона защиты

Б) следователь 2) сторона обвинения

В) обвиняемый

Г) потерпевший

Д) защитник

14. Выберите верные суждения об отклоняющемся поведении

1) отклоняющееся поведение индивида всегда связано с нарушением нравственных норм 2) отклоняющееся поведение индивида может быть вызвано его социальным окружением 3) отклоняющееся поведение индивида может иметь как негативный характер. Так и позитивный 4) отклоняющееся поведение проявляется только у индивидов и не свойственно социальным группам 5) отклоняющееся поведение влечет определенные социальные санкции

15. В государстве существует наследственная передача власти. Власть короля ограничена законами страны. Выборы в парламент происходят регулярно на альтернативной основе. Граждане обладают всей полнотой прав и свобод. Развиты институты гражданского общества. Государство В. включает территории субъектов, обладающих частичным суверенитетом. Парламент имеет двухпалатную структуру, субъекты вправе принимать собственные конституции. Найдите в списке характеристики формы государства В. 1) унитарное государство

2) федеративное государство

3) конституционная монархия

4) демократическое государство

5) абсолютная монархия

6) президентская республика

16. Установите соответствие между примерами и видами потребностей

ПРИМЕРЫ ВИДЫ ПОТРЕБНОСТЕЙ

А) в общении 1) духовные

Б) в приобретении новых знаний 2) социальные

В) в пище 3) биологические

Г) в общественном признании

Д) в воздухе для дыхания

17. Анна Петровна активный и творческий человек. Она выступила инициатором проведения субботника в своём дворе. Это характеризует её как:

1)личность;         2) интеллигентного человека; 3)администратора;          4) профессионала;

18. Установите соответствие между отличительными признаками и типами обществ.

ОТЛИЧИТЕЛЬНЫЕ ПРИЗНАКИ ТИПЫ ОБЩЕСТВ

А) основополагающая роль религии 1) аграрное

Б) сословная структура общества 2) индустриальное

В) серийное производство товаров массового потребления 3) постиндустриальное

Г) выдвижение на первый план сферы услуг

Д) ведущая роль информации в жизни общества

19. На графике отражена ситуация на рынке кондитерских изделий: кривая предложения переместилась из положения S в положениеS1. Какие из перечисленных факторов могут вызвать такое изменение?

1) неурожай какао-бобов 2) увеличение доходов потребителей 3) выдача льготных кредитов кондитерским предприятиям 4) повышение акцизов на сахар и какао-бобы 5) закрытие ряда кондитерских фабрик

20. А. был избран губернатором области. Он уделяет большое внимание защите прав и свобод граждан, развитию институтов гражданского общества. В общении он доброжелателен. Открыт для критики. Что из перечисленного характеризует тип политического лидерства. Запишите цифры, под которыми указаны верные характеристики

1) общенациональный 2) демократический 3) традиционный 4) авторитарный 5) региональный 6) харизматический

21. Вставьте необходимые слова на место пропусков.

Социальные нормы — ___________(А), регулирующие взаимоотношения людей и их объединений. Общеобязательные, формально-определенные правила поведения, которые установлены или санкционированы, а также охраняются государством, называются __________(Б). Сложившиеся в обществе правила поведения, которые выражают представления людей о добре и зле, справедливости и несправедливости, долге называются __________(В). Действие этих норм обеспечивается внутренним убеждением, ____________-(Г), мерами общественного воздействия. В результате многократного повторения, в силу привычки исполняются _____(Д). есть и иные виды социальных норм, и все они выполняют роль регулятора __________(Е).

Список терминов:

1)общественное мнение 2) общественное отношение

3) социальные группы

4) корпоративные нормы

5) социальная динамика

6) нормы обычаев

7) правила поведения

8) нормы морали

9) нормы права

Входные тестовые задания 11 класс Обществознание.

Вариант III.

  1. Запишите пропущенное слово

Области духовной культуры

Области духовной культуры

Характеристики

Искусство

Отражение мира и человека в художественных образах

……………….

Целенаправленный процесс обучения и воспитания

  1. Найдите понятие, которое является обобщающим для всех остальных понятий и запишите цифру, под которой оно указано

познание 2) игра 3)труд 4)общение 5) деятельность

3. Ниже приведен ряд терминов. Все они, за исключением двух, представляют черты элитарной культуры. Найдите два термина и запишите их номера.

1) отсутствие выраженной коммерческой направленности 2) доступность для массового зрителя 3) простота 4)стремление творца к самовыражению 5) сложность художественных приемов 6) узкий круг потребителей

4. Выберите верные суждения об обществе и его типах и запишите цифры, под которыми они указан

1) Общество – это совокупность всех форм объединения и способов взаимодействия людей, в которых выражается их взаимозависимость 2) основной фактор производства индустриального общества – земля 3) В узком смысле под обществом понимают определенный этап исторического развития. 4) В обществе традиционного типа экономика носит аграрно-сырьевой характер, господствует натуральное хозяйство, преобладает ручной труд 5) Общество представляет собой закрытую систему, но взаимодействует с внешней средой.

5. Установите соответствие между формами и этапами познания

ФОРМЫ ПОЗНАНИЯ ЭТАПЫ ПОЗНАНИЯ

А) суждение 1) чувственное познание

Б) представление 2) рациональное познание

В) восприятие

Г) понятие

Д) ощущение

6. . Популярный актер и телеведущий на собственные средства снял черно-белый фильм о судьбе своего поколения. Критики отнесли это произведение к элитарному искусству. Запишите цифры, под которыми указаны характеристики произведения элитарной культуры..

1) коммерческий характер 5) привлечение кинозвезд 2) содержательная сложность 6) большой интерес зрителей 3) внимание критиков и знатоков 4) использование спецэффектов

7.. Найдите в приведенном списке признаки, позволяющие сделать вывод о том, что страна В развивается как постиндустриальное общество

1) в стране производство промышленных товаров механизировано 2) в стране развиваются наукоемкие производства и средства коммуникации 3) в стране созданы необходимые условия для непрерывного образования 4) в стране доступ к информации является одним из условий успеха в конкурентной борьбе 5) в стране граждане добились равноправия и избирательных прав 6) в стране активно развивается виртуальная культура

8. В стране , расположенной на морском побережье, активно развивается туристическая отрасль. Какие признаки подтверждают наличие в стране рыночной экономики?

1) В стране каждый собственник факторов производства свободно распоряжается ими 2) В стране производится широкий ассортимент товаров народного потребления 3) Собственниками земли, предприятий являются частные лица 4) Правительство вкладывает значительные средства в строительство дорог 5) В столице государства открыт новый международный аэропорт 6) В стране существует конкуренция производителей

9.Установите соответствие

Характеристики Факторы производства

А) все виды ресурсов, имеющихся на планете и пригодных 1) труд для производства экономических благ 2) земля

Б) деятельность людей по производству товаров и услуг 3) капитал путем использования их физических и интеллектуальных возможностей, навыков и опыта

В) факторный доход – заработная плата

Г) здания, сооружения, оборудование

Д) объем денежной массы

10. Выберите верные суждения о социальной мобильности

1) К горизонтальному виду мобильности относится получение офицером внеочередного воинского звания 2) К вертикальному виду мобильности относится переход человека в более низкий социальный статус 3) Социальная мобильность – это разделение общества на группы, занимающие разное социальное положение 4)Межпоколенная мобильность — -сравнительное изменение социального статуса у разных поколений.

11. В стране К правительство формируется блоком политических партий, победившим на парламентских выборах. Найдите в списке черты, свидетельствующие, что в стране К выборы парламента проходят по пропорциональной системе

1) Голосование проводится по спискам политических партий 2) Существует возможность выдвижения независимых беспартийных кандидатов 3) Граждане голосуют прежде всего за программы партий, а не за конкретных людей. 4) Количество мест, полученных партией в парламенте зависит от процента голосов, поданных за партию на выборах. 5) Победу одерживает кандидат, получивший большинство голосов на выборах 6) Предусмотрено голосование по одномандатным округам.

12. Что из перечисленного относится к конституционным обязанностям гражданина РФ?

1) сохранение исторического и культурного наследия 2) уплата законно установленных налогов и сборов 3) защита Отечества 4) участие в управлении делами государства 5) выбор профессии

13. Установите соответствие

СУБЪЕКТЫ УЧАСТНИКИ УГОЛОВНОГО СУДОПРОИЗВОДСТВА

А) прокурор 1) сторона защиты

Б) следователь 2) сторона обвинения

В) обвиняемый

Г) потерпевший

Д) защитник

14. Выберите верные суждения об отклоняющемся поведении

1) отклоняющееся поведение индивида всегда связано с нарушением нравственных норм 2) отклоняющееся поведение индивида может быть вызвано его социальным окружением 3) отклоняющееся поведение индивида может иметь как негативный характер. Так и позитивный 4) отклоняющееся поведение проявляется только у индивидов и не свойственно социальным группам 5) отклоняющееся поведение влечет определенные социальные санкции

15. В государстве Н. существует наследственная передача королевской власти. Регулярно на альтернативной основе проходят выборы в парламент, выполняющий законодательные функции. Граждане обладают всей полнотой прав и свобод, развиты институты гражданского общества. Госсударство Н. включает в себя территориальные единицы, не обладающие политической самостоятельностью. Найдите в списке характеристики формы государства.

1) конституционная монархия 2) федеративное государство 3) унитарное государство 4) демократическое государство 5) абсолютная монархия 6) президентская республика

16. Установите соответствие между примерами и видами потребностей

ПРИМЕРЫ ВИДЫ ПОТРЕБНОСТЕЙ

А) в общении 1) духовные

Б) в приобретении новых знаний 2) социальные

В) в пище 3) биологические

Г) в общественном признании

Д) в воздухе для дыхания

17. Ларисе 17 лет. Найдите в списке черты, имеющие социальный характер

1) Рост Ларисы ниже среднего  

2) Лариса – честный человек

3) У Ларисы светлые волосы и зеленые глаза.

4) Лариса добрая и отзывчивая.

5)Лариса дружит со своими одноклассниками

6)Лариса   внешне привлекательная девушка                            

18. Установите соответствие между отличительными признаками и типами обществ.

ОТЛИЧИТЕЛЬНЫЕ ПРИЗНАКИ ТИПЫ ОБЩЕСТВ

А) сословная структура общества 1) аграрное

Б) ведущая роль информации в жизни общества 2) индустриальное

В) серийное производство товаров массового потребления 3) постиндустриальное

Г) выдвижение на первый план сферы услуг

Д) утверждение ценностей личного успеха. Прогресса

19. На графике отражено изменение спроса на школьную форму: кривая спроса переместилась из положения Д в положение Д1. Какие из перечисленных факторов могут вызвать такое изменение?

1) снижение доходов потребителей

2) принятие закона об обязательной школьной форме

3) увеличение таможенных пошлин на импортные ткани

4) повышение цен на энергоносители

5) скоро начало нового учебного года в школах

20. А. был избран губернатором области. Он уделяет большое внимание защите прав и свобод граждан, развитию институтов гражданского общества. В общении он доброжелателен. Открыт для критики. Что из перечисленного характеризует тип политического лидерства. Запишите цифры, под которыми указаны верные характеристики

1) общенациональный 2) демократический 3) традиционный 4) авторитарный 5) региональный 6) харизматический

21. Вставьте необходимые слова на место пропусков.

Социальные нормы — ___________(А), регулирующие взаимоотношения людей и их объединений. Общеобязательные, формально-определенные правила поведения, которые установлены или санкционированы, а также охраняются государством, называются __________(Б). Сложившиеся в обществе правила поведения, которые выражают представления людей о добре и зле, справедливости и несправедливости, долге называются __________(В). Действие этих норм обеспечивается внутренним убеждением, ____________-(Г), мерами общественного воздействия. В результате многократного повторения, в силу привычки исполняются _____(Д). есть и иные виды социальных норм, и все они выполняют роль регулятора __________(Е).

Список терминов:

1)общественное мнение 2) общественное отношение

3) социальные группы

4) корпоративные нормы

5) социальная динамика

6) нормы обычаев

7) правила поведения

8) нормы морали

9) нормы права

Контрольная работа по обществознанию 10 класса (УМК Л.Н.Боголюбов) по «Общество и человек»

1.Определение «Совокупность исторически сложившихся форм совместной деятельности людей» относится к понятию» 1. Цивилизация 2.Общество 3. Класс 4.Формация

2.Определение «Результат целенаправленной и разумно организованной совместной деятельности людей, объединенных на основе общих интересов» относится к понятию:

1.Цивилизация 2.Общество 3. Класс 4.Формаци

3. Человек, согласно современным представлениям, есть существо

1) духовное 2) социальное 3) биологическое 4) биосоциальное

4.  Иван — высокий, худощавый, с красивыми чертами лица, мужественный, расчётливый, медлительный и осторожный. Всё это характеризует Ивана как

1) личность 2) гражданина 3) индивидуальность 4) профессионала

5. Верны ли следующие суждения о сходстве и различии человека и животного?

А. Муравьи и другие «социальные» животные трудятся так же, как и люди.

Б. Все особи животных, в отличие от людей, всегда действуют согласно генетической программе.

1) верно только А3) верны оба суждения

2) верно только Б4) оба суждения неверны

6. Какой признак характеризует человека как личность? 1) нейродинамические свойства мозга

2) психофизиологические функции 3) непринуждённость в деятельности 4) жизненная позиция

7. Определение: «Совокупность идей, взглядов, теорий, а также чувств, привычек и нравов определённой социальной общности или группы» относится к понятию

1) общественное сознание 2) общество 3) обыденное сознание 4) идеология

8.  Определение: «Обобщённая система взглядов человека на мир в целом, на своё собственное место в нём, понимание и оценка человеком смысла своей жизни и деятельности» относится к понятию

1) менталитет 2) мировоззрение 3) сознание 4) мироощущение

9. К религиозному мировоззрению не относится

1) тесная связь с мировым культурным наследием

2) стремление дать человеку веру в возможность достижения поставленных целей

3) ориентация на решение проблем, связанных с духовными потребностями человека

4) формулирование совокупности рационально построенных прогнозов

10. И чувственное, и рациональное познание:

1) формирует представления и знания о предмете

2) начинается с ощущения

3) даёт наглядный образ предмета

4) использует логическое умозаключение

11. Какому знанию соответствуют следующие выражения: «на глазок», «чуть-чуть», «щепоточка»?

1) научному 2) художественному 3) практическому 4) житейскому

12. Верны ли следующие суждения о познании?

А. Познание относится к базовым потребностям человека.

Б. Познание всегда носит творческий характер.

1) верно только А3) верны оба суждения

2) верно только Б4) оба суждения неверны

13. Утверждение: «Товар имеет стоимость» является примером

1) представления3) суждения

2) понятия4) умозаключения

14. Знание, соответствующее своему предмету, совпадающее с ним, есть

1) закон2) проблема 3) истина4) принцип

15. Утверждение: «Гавайские острова лежат в Тихом океане» является примером

1) относительной истины 2) заблуждения 3) субъективной истины 4) абсолютной истины

16. Верны ли следующие суждения об истине?

А. Абсолютной истиной называют знание, с которым все согласны, поскольку это то, что очевидно, что нельзя представить иначе.

Б. Относительная истина представляет собой знание, достаточное для успешного ведения человеком своих дел.

1) верно только А3) верны оба суждения

2) верно только Б4) оба суждения неверны

17. Относительность истины заключается

1) в неполноте научных теорий

2) в субъективности формы познания истины

3) в приблизительности суждений «истина и ложь»

4) во всём перечисленном

18. Основа человеческого существования — это

1) дружба3) потребительство

2) любовь4) деятельность

19. Верны ли следующие суждения о деятельности человека?

А. Деятельность человека выражается в приспособлении к природной среде путём её масштабного преобразования, ведущего к созданию искусственной среды существования.

Б. Деятельность человека предполагает сознательную постановку целей, связанных со способностью анализировать ситуацию.

1) верно только А3) верны оба суждения

2) верно только Б4) оба суждения неверны

20. К осмысленным побудителям деятельности человека относятся

1) привычки3) мотивы

2) влечения4) эмоции

Индивидуальные проверочные работы по обществознанию (10-й класс)

В данной публикации Вашему вниманию я предлагаю:

  • Две индивидуальные практические работы по обществознанию в 10 классе. Задания основаны на УМК Л.Н. Боголюбова. Их можно использовать на итоговых, повторительных уроках в конце учебного года. Я преподаватель спортивной школы, учащиеся у нас часто отсутствуют, данные работы позволяют качественно повторить и закрепить материал. Каждая проверочная работа состоит из заданий по пяти основным, изученным в 10 классе, разделам. В него включены вопросы по теории и задания по применению знаний на практике. За каждый блок заданий выставляется три оценки: одна – за знание понятий и основных терминов; вторая — за работу с источниками и документами; третья – за тестовую работу.
  • Итоговую тестовую работу по обществознанию за курс 7 класса по УМК А. И. Кравченко. Общее число заданий в работе – 20. Работа состоит из 2 – х частей:

Часть 1 (А) содержит 14 заданий с выбором ответа (один верный ответ из предложенных). С их помощью проверяются базовые знания понятий и терминов, умения описывать и сравнивать основные социальные объекты, выделяя их существенные признаки. Оценка каждого задания этой группы – 1 балл.

Часть 2 (В) состоит из 6 заданий с открытым кратким ответом: слово, название документа. Эти задания позволяют проверить умения извлекать информацию из источника, классифицировать и систематизировать факты, решать в рамках изученного материала познавательные и практические задачи, отражающие типичные ситуации в различных сферах деятельности человека; Все задания этой группы оцениваются в соответствии с уровнем сложности: 2 задания — по 1 баллу каждое, 3 задания – по 2 балла.

Структура и содержание

Распределение заданий работы по частям работы

Часть работы Число заданий Максимальный первичный балл % максимального первичного балла за задания данной части от максимального первичного балла за всю работу Тип заданий
Часть 1 (А) 14 14 70% С выбором ответа
Часть 2 (В) 6 9 30% С кратким ответом
Итого: 20 23 100  

Распределение заданий по основным разделам курса

Разделы курса обществознания Число заданий Максимальный первичный балл % максимального первичного балла за задания данной части от максимального первичного балла за всю работу
Личность подростка А2, 3, 14,В4, 6 5 6 26
Подросток в социальной среде А7,8,13 3 3 13.4
Подросток и закон А4,5,6,10, В1,3 6 7 30.4
Образ жизни подростков А1,9,11,12 4 4 17,3
Подросток и его жилая среда В2,5 2 3 13.4
Итого: 20 23 100

Распределение заданий по уровню сложности

Уровень сложности заданий Число заданий Максимальный первичный балл % максимального балла за задания данного уровня сложности от максимального первичного балла за всю работу
Базовый 15 15 65,2
Повышенный 5 8 34,8
Итого: 20 23 100

Распределение заданий по видам проверяемых знаний и умений

Виды знаний и умений Число заданий Максимальный первичный балл % максимального балла за задания данного уровня сложности от максимального первичного балла за всю работу
Группировка фактов 3 3 13
Знание терминов 5 5 21,7
Знание понятий 1 1 4,3
Соотнесение единичных фактов и общих явлений; указанных характерных признаков событий и явлений 1 2 8,6
На установление последовательности 1 1 4,3
Объяснение причин и следствий 2 2 8,6
Задания на обобщенную характеристику, систематизацию изученного материала. Проверяется комплекс знаний и умений 4 6 26
Систематизация фактов, понятий 2 2 8,6
Установление предмета по признакам 1 1 4,3
Итого: 20 23 100

Система оценивания отдельных заданий и работы в целом. Баллы:

Оценивание каждого задания части 1 (А) – по 1 баллу.

Оценивание заданий части (В):

Каждое из заданий: В1, В2 , В3 – по 1 баллу;

Каждое из заданий: В4, В5, В6 – по 2 балла

Максимальный первичный балл за всю работу – 23 балла

Шкала перевода баллов в оценки

Оценка

“2”; 50%

“3”; 75-85%

“4”; 90-95%

“5”; 95–100%

Соотношение баллов, % выполнения работы Менее 12 баллов 16-18 баллов 19-20 баллов 20 – 23 баллов

Сводная таблица распределения заданий по основным разделам курса,
по уровню сложности, по видам проверяемых знаний и умений

№ п/п Обозначение задания в работе Проверяемое содержание – раздел курса Проверяемые виды деятельности Уровень сложности задания Тип задания Макс. балл за выполнение задания
1 А – 1 Образ жизни подростков Знание терминов Б ВО

Диагностическая работа по обществознанию. Входной контроль. — Студопедия.Нет

Диагностическая работа по обществознанию. Входной контроль.

Класс (профильный уровень).

Сентябрь 2017 г.

Спецификация для диагностической работы по обществознанию.

Время выполнения диагностической работы:

На выполнение всей диагностической работы отводится 45 минут.

Учебники:

Боголюбов Л.Н., Лазебникова А.Ю., Смирнова Н.М. и др. под ред. Боголюбова Л.Н., Лазебниковой А.Ю. Обществознание (профильный уровень) 10 класс М.Просвещение, 2012.

Экономика. Основы экономической теории. Учебник для 10-11 кл. общеобразоват. учрежд. Профильный уровень образования /Под ред. С. И. Иванова. — 15-е изд. — В 2-х книгах. Книга 1. — М.: ВИТА-ПРЕСС, 2012.

Никитин, А. Ф. Право. 10-11 кл. Учебник для общеобразоват. учреждений / А. Ф. Никитин. — 4-е изд., перераб. — М.: Дрофа, 2013.

Проверяемые элементы содержания:

Человек и общество

1.1 Природное и общественное в человеке. (Человек как результат биологической и социокультурной эволюции)

1.3 Виды знаний

1.4 Понятие истины, её критерии

1.5 Мышление и деятельность

1.6 Потребности и интересы

1.7 Свобода и необходимость в человеческой деятельности. Свобода и ответственность

1.8 Системное строение общества: элементы и подсистемы

1.9 Основные институты общества

1.16 Понятие общественного прогресса

1.17 Многовариантность общественного развития (типы обществ)

Экономика

2.1 Экономика и экономическая наука

2.2 Факторы производства и факторные доходы

2.3 Экономические системы

2.4 Рынок и рыночный механизм. Спрос и предложение

2.5 Постоянные и переменные затраты

2.6 Финансовые институты. Банковская система

2.7 Основные источники финансирования бизнеса

2.8 Ценные бумаги

2.16 Рациональное экономическое поведение собственника, работника, потребителя, семьянина, гражданина

Социальные отношения

3.2 Социальные группы

3.3 Молодёжь как социальная группа

3.7 Социальный конфликт

3.10 Семья и брак

3.13 Социализация индивида

Право

5.1 Право в системе социальных норм

5.2 Система российского права. Законотворческий процесс

5.3 Понятие и виды юридической ответственности

5.4 Конституция Российской Федерации. Основы конституционного строя Российской Федерации

5.5 Законодательство Российской Федерации о выборах

5.12 Право на благоприятную окружающую среду и способы его защиты

5.13 Международное право (международная защита прав человека в условиях мирного и военного времени)

5.17 Гражданство Российской Федерации

5.18 Воинская обязанность, альтернативная гражданская служба

5.20 Правоохранительные органы. Судебная система

Структура диагностической работы:

Каждый вариант диагностической работы содержит 20 заданий с кратким ответом. Ответами к заданиям являются слово (словосочетание) или последовательность цифр.

Задания 1–3, 10, 12 оцениваются 1 баллом. Задание считается выполненным верно, если ответ записан в той форме, которая указана в инструкции по выполнению задания.

Правильное выполнение заданий 4–9, 11, 13–20 оценивается 2 баллами. Эти задания оцениваются следующим образом: полное правильное выполнение задания – 2 балла; выполнение задания с одной ошибкой (одной неверно указанной, в том числе лишней, цифрой наряду со всеми верными цифрами) ИЛИ неполное выполнение задания (отсутствие одной необходимой цифры) – 1 балл; неверное выполнение задания (при указании двух или более ошибочных цифр) – 0 баллов.

Максимальное количество баллов – 35

 

Диагностическая работа по обществознанию. Входной контроль.

Класс ( профильный уровень )

45 мин.                                                                                                                            Сентябрь 2017 г.

Вариант 1.

1. Запишите слово, пропущенное в таблице.

ТИП СЕМЬИ ХАРАКТЕРИСТИКА
Расширенная Семья, состоящая из живущих вместе двух и более поколений
  ************ Семья, состоящая из супружеской четы с детьми или без детей, или одного из родителей со своими детьми

2. В приведённом ниже ряду найдите понятие, которое является обобщающим для всех остальных представленных понятий. Запишите это слово (словосочетание).

Реформа; революция; социальная динамика; эволюция; общественный регресс.

3. Ниже приведён перечень признаков. Все они, за исключением двух, характеризуют постиндустриальное общество.

1) Развитие культуры; 2) компьютеризация и роботизация; 3) права человека; 4) человек оценивается как часть коллектива; 5) демократизация политической жизни; 6) укрепление сословного строя.

4. Выберите верные суждения об обществе и его институтах и запишите цифры, под которыми они указаны.

1) В узком смысле общество — это окружающий человека материальный мир.

2) В широком смысле под обществом понимают всё население Земли, совокупность всех народов и стран.

3) Общество является самоорганизующейся системой.

4) Динамичность социальных институтов общества проявляется в их обособленности от природы.

5) Социальный институт — это исторически сложившаяся устойчивая форма организации совместной деятельности, направленной на удовлетворение базовых потребностей общества.

5. Установите соответствие между примерами и видами потребностей человека: к каждой позиции, данной в первом столбце, подберите соответствующую позицию из второго столбца.

ПРИМЕРЫ А) в пище Б) в общении В) в приобретении новых знаний Г) в общественном признании Д) в воздухе для дыхания ВИДЫ ПОТРЕБНОСТЕЙ 1) духовные (идеальные) 2) социальные 3) биологические (естественные)

6. Студент Пётр Иванов готовится к сдаче экзамена. Выберите из предложенного перечня ситуации, в которых он выступал как субъект познавательной деятельности. Запишите цифры, под которыми они указаны.

1) Выбрал учебные пособия для подготовки.

2) Сильно волновался в ночь перед экзаменом.

3) Забыл прийти на предэкзаменационную консультацию.

4) Подготовил краткие конспекты по основным вопросам.

5) С одногруппниками вслух проговорил ключевые положения каждого ответа.

6) Чтобы снять стресс, слушал накануне экзамена лёгкую инструментальную музыку.

7. Выберите верные суждения об экономическом развитии и запишите цифры, под которыми они указаны.

1) На экономическое развитие страны оказывают влияние исторические и географические условия её существования.

2) Для экономического развития разных стран не существует единых закономерностей.

3) Одним из показателей экономического развития общества является отраслевая структура экономики.

4) Для экономического развития характерна как положительная динамика экономики, так и отрицательная.

5) Объём ВВП на душу населения характеризует экономический потенциал страны.

8. Установите соответствие между источниками финансирования бизнеса и типами источников: к каждой позиции, данной в первом столбце, подберите соответствующую позицию из второго столбца.

ИСТОЧНИК ФИНАНСИРОВАНИЯ БИЗНЕСА А) чистая прибыль Б) банковский кредит В) амортизационные отчисления Г) средства внебюджетного фонда Д) средства населения ТИП ИСТОЧНИКА 1) внутренние источники финансирования бизнеса 2) внешние источники финансирования бизнеса

9. В стране Z был принят закон, регулирующий передачу крупных предприятий в частную собственность. Какие дополнительные факторы позволят сделать вывод о том, что в стране Z осуществляется переход к рыночной экономической системе? Запишите цифры, под которыми они указаны.

1) В стране Z был повышен размер минимальной оплаты труда.

2) В стране Z была отменена уголовная ответственность за покупку иностранной валюты.

3) В стране Z значительно вырос реальный ВВП на душу населения.

4) Государственный бюджет страны Z стал публиковаться в открытых источниках.

5) В конституции страны Z было закреплено существование различных форм собственности.

6) В стране Z частные инвесторы стали привлекаться к разработке мест полезных ископаемых.

10. На графике изображено изменение предложения саженцев плодово-ягодных растений на потребительском рынке: кривая предложения переместилась из положения S в положение S1. (На графике P – цена товара; Q – количество товара.)

Какие из перечисленных факторов могут вызвать такое изменение? Запишите цифры, под которыми они указаны.

1) наступление весеннего сезона

2) рост числа агрофирм

3) увеличение цен на минеральные удобрения

4) увеличение поставок импортных фруктов и ягод на потребительский рынок

5) развитие системы льготного кредитования приусадебных хозяйств

11. Выберите верные суждения о социальных группах и запишите цифры, под которыми они указаны.

1) Социальной группой называется совокупность людей, объединяемая устойчивыми социальными связями и отношениями и обладающая рядом признаков, придающих ей неповторимое своеобразие.

2) Принадлежность к социальной группе помогает человеку осознать своё положение в обществе.

3) Русские, белорусы, немцы — социальные группы, выделенные по социально-демографическому признаку.

4) Принадлежность к различным социальным группам определяет профессию человека.

5) В формальной группе статус членов и цель деятельности не всегда чётко определены, взаимодействия в ней основаны на взаимной симпатии, общем интересе или привычке.

12. Социологические службы страны Z провели опрос среди граждан, в ходе которого им задавался вопрос: «Был ли такой период в Вашей жизни, когда Вы ощущали себя бедным? Если да, то когда?» (Возможно несколько вариантов ответа.) Полученные результаты опроса представлены в виде таблицы.

Варианты ответа 1990 г. 2015 г.
Всегда ощущал себя бедным 28 16
В первые годы самостоятельной жизни 34 30
В детстве 11 19
Когда учился в техникуме, вузе, ПТУ 27 28
Когда вышел на пенсию 31 22
Когда в жизни случилось несчастье 6 4
Такого периода в моей жизни не было 3 26

Найдите в приведённом списке выводы, которые можно сделать на основе таблицы, и запишите цифры, под которыми они указаны.

1) За последнюю четверть века социальное самочувствие жителей страны Z изменилось в лучшую сторону.

2) Обычно люди чувствуют себя бедными, когда в их жизни происходит несчастье.

3) Доля тех, кто связывает ощущение бедности с выходом на пенсию, увеличилась.

4) Среди причин, вызывающих ощущение бедности, устойчиво лидирует начало самостоятельной жизни.

5) В период получения профессии меньше людей чувствуют себя бедными, чем в период детства.

13. Выберите верные суждения о молодежи как социальной группе и запишите цифры, под которыми они указаны.

1) Для молодежи характерна собственная субкультура.

2) Молодежь как социальная группа выделяется на основе возрастного критерия.

3) Большинству молодых присущ более высокий социальный статус, чем людям зрелого возраста.

4) В отличие от подростков ведущей деятельностью молодых является познание.

5) Молодежи свойственно стремление к социальному самоопределению.

14. Установите соответствие между полномочиями и субъектами государственной власти РФ, реализующими эти полномочия: к каждой позиции, данной в первом столбце, подберите соответствующую позицию из второго столбца.

ПОЛНОМОЧИЯ А) утверждение изменения границ между субъектами РФ Б) осуществление управления федеральной собственностью В) обеспечение исполнения федерального бюджета Г) утверждение военной доктрины РФ Д) назначение на должность судей Конституционного Суда РФ СУБЪЕКТЫ ГОСУДАРСТВЕННОЙ ВЛАСТИ 1) Совет Федерации 2) Президент РФ 3) Правительство РФ

15. Выберите верные суждения о военной службе и альтернативной гражданской службе. Запишите цифры, под которыми они указаны.

1) Воинская обязанность — установленный законом долг граждан нести службу в рядах Вооружённых Сил и выполнять другие обязанности, связанные с обороной страны.

2) На военную службу призываются все граждане РФ в возрасте от 18 до 30 лет.

3) Граждане могут быть освобождены от исполнения воинской обязанности только по основаниям, предусмотренным законом.

4) Гражданин имеет право на замену военной службы по призыву альтернативной гражданской службой, если несение военной службы противоречит его убеждениям или вероисповеданию.

5) Срок альтернативной гражданской службы в 1,75 раза меньше установленного срока военной службы по призыву.

16. Что из перечисленного относится к личным (гражданским) правам гражданина РФ? Запишите цифры, под которыми они указаны.

1) право на социальное обеспечение 2) право на жизнь 3) право на жилище 4) право на защиту чести и доброго имени 5) право на свободу и личную неприкосновенность

17. Выберите верные суждения о судопроизводстве в РФ и запишите цифры, под которыми они указаны.

1) Судебная власть в РФ осуществляется посредством конституционного, гражданского, административного и уголовного судопроизводства.

2) Судопроизводство — это деятельность судов по рассмотрению дел.

3) Конституционное судопроизводство — это рассмотрение и разрешение гражданских дел в целях защиты нарушенных или оспариваемых прав, свобод и законных интересов субъектов гражданских, семейных, трудовых или иных правоотношений.

4) Гражданское судопроизводство осуществляется на основе состязательности и равноправия сторон.

5) Сторонами в уголовном судопроизводстве являются истец и ответчик.

18. Установите соответствие между правами и юридическим статусом человека: к каждой позиции, данной в первом столбце, подберите соответствующую позицию из второго столбца.

ПРАВО A) на свободу слова Б) на участие в работе суда в качестве присяжного заседателя B) на участие в управлении государством Г) на свободу митингов Д) на владение частной собственностью ЮРИДИЧЕСКИЙ СТАТУС 1) гражданин РФ 2) все лица, проживающие на территории РФ

19. Найдите в приведенном списке правоохранительные органы РФ и запишите цифры, под которыми они указаны.

1) Государственная Дума 2) Администрация Президента 3) прокуратура 4) полиция 5) Совет безопасности 6) Министерство иностранных дел

20. Прочитайте приведённый ниже текст, в котором пропущен ряд слов. Выберите из предлагаемого списка слова, которые необходимо вставить на место пропусков.

«Человек, активно осваивающий и целенаправленно преобразующий природу, общество и самого себя, является *** (А). Это человек со своими социально сформированными и с индивидуально выраженными *** (Б): интеллектуальными, эмоционально-волевыми, нравственными и др. Их формирование связано с тем, что *** (В) в совместной деятельности с другими людьми познаёт и изменяет мир и самого себя. Процесс этого познания в ходе усвоения и воспроизводства социального опыта одновременно является процессом *** (Г). Личность определяют как особую форму существования и развития социальных связей, отношений к миру и с миром, к себе и с самим собой. Она характеризуется *** (Д), т.е. стремлением выходить за свои собственные пределы, развиваться, расширять сферу своей деятельности, открыта всем влияниям общественной жизни, всякому опыту. Это человек, у которого есть своя *** (Е) в жизни, который проявляет самостоятельность мысли, несёт ответственность за свой выбор».

Слова в списке даны в именительном падеже. Каждое слово может быть использовано только один раз. Выбирайте последовательно одно слово за другим, мысленно заполняя каждый пропуск. Обратите внимание на то, что слов в списке больше, чем Вам потребуется для заполнения пропусков.

Список терминов:

1) индивид 2) качество 3) потребность 4) воспитание 5) позиция 6) социализация 7) личность 8) активность 9) индивидуальность

 

 

Диагностическая работа по обществознанию. Входной контроль.

Класс (профильный уровень)

45 мин.                                                                                                                            Сентябрь 2017 г.

Вариант 2.

1. Запишите слово, пропущенное в таблице.

НАПРАВЛЕНИЕ ОБЩЕСТВЕННОГО РАЗВИТИЯ ХАРАКТЕРИСТИКА
Прогресс Направление поступательного развития, для которого характерен переход от низшего к высшему, от менее совершенного к более совершенному
  *********** Переход от более высоких форм развития к более низким; упадок в развитии

2. В приведенном ниже ряду найдите понятие, которое является обобщающим для всех остальных представленных понятий. Запишите это слово (словосочетание).

Умозаключение; формы познания; ощущение; гипотеза; представление.

3. Ниже приведён перечень признаков. Все они, за исключением двух, характеризуют индустриальное общество.

1) Промышленное производство; 2) компьютеризация и роботизация; 3) права человека; 4) массовая культура; 5) демократизация политической жизни; 6) укрепление сословного строя. Найдите и запишите номера признаков, выпадающих из этого ряда.

4. Выберите верные суждения о человеке и запишите цифры, под которыми они указаны.

1) К духовным (идеальным) потребностям человека традиционно относят потребности в воздухе, питании, поддержании нормального теплообмена.

2) К естественным (биологическим) потребностям человека относят потребности в познании окружающего мира, достижении гармонии и красоты; религиозной вере, художественном творчестве и т.п.

3) Деятельность — это специфический способ существования человека.

4) Потребности — это переживание человеком нужды в том, что необходимо для поддержания жизни и развития личности.

5) Только человек способен сознательно преобразовывать окружающую действительность, создавать необходимые ему блага и ценности.

5. Установите соответствие между отличительными признаками и видами деятельности, которые они иллюстрируют: к каждой позиции, данной в первом столбце, подберите соответствующую позицию из второго столбца.

ОТЛИЧИТЕЛЬНЫЙ ПРИЗНАК А) преобразование природы и общества ради удовлетворения личных и социальных потребностей Б) направленность на практически полезный результат – различные блага В) формирование знаний и умений, развитие мышления и сознания личности Г) побудительный мотив заключается не в результате деятельности, а в её процессе Д) все средства всегда специально направлены на изменение субъекта деятельности ВИД ДЕЯТЕЛЬНОСТИ 1) труд 2) игра 3) учёба

6. Учёный-биолог Петров изучает роль ядовитых грибов в жизни леса. Выберите из приведённого перечня эмпирические методы исследования, используемые Петровым. Запишите цифры, под которыми они указаны.

1) выдвинул гипотезу о защитных функциях ядовитых грибов по отношению к некоторым видам растений

2) установил в лаборатории химический состав нескольких видов ядовитых грибов Подмосковья

3) изготовил модель развития грибницы в различных природных условиях

4) подготовил список основной литературы по изучаемой проблеме

5) подготовил иллюстрированный атлас с описанием наиболее распространённых видов ядовитых грибов

6) зафиксировал с помощью видеозаписи ареал распространения основных видов ядовитых грибов Подмосковья

7. Выберите верные суждения об источниках финансирования бизнеса и запишите цифры, под которыми они указаны.

1) Наращивание объёмов внешнего финансирования бизнеса повышает степень контроля собственника за предприятием.

2) Наиболее распространённой формой финансирования является банковский кредит.

3) Внутреннее финансирование бизнеса не сопряжено с дополнительными расходами, связанными с привлечением капитала.

4) К внешним источникам финансирования бизнеса относится сдача в аренду неиспользуемых активов фирмы.

5) Финансирование частного бизнеса не может носить государственный характер.

8. Установите соответствие между характеристиками и видами ценных бумаг: к каждой позиции, данной в первом столбце, подберите соответствующую позицию из второго столбца.

ХАРАКТЕРИСТИКА ЦЕННЫХ БУМАГ А) ценная бумага, закрепляющая право её держателя на получение от эмитента в предусмотренный ею срок номинальной стоимости и зафиксированного в ней процента от этой стоимости или имущественного эквивалента Б) ценная бумага, в которой содержится распоряжение плательщика своему банку уплатить получателю указанную в ней сумму денег в течение срока её действия В) держатель этой ценной бумаги имеет право на часть имущества, остающегося после ликвидации предприятия Г) денежный документ, удостоверяющий внесение средств на определённое время, имеющий обычно фиксированную ставку процента Д) ценная бумага, из суммы номиналов которой складывается уставный капитал коммерческой организации ВИД ЦЕННЫХ БУМАГ 1) акция 2) облигация 3) сберегательный сертификат 4) чек

9. В стране N командный тип экономической системы. Какие факты из приведённого списка свидетельствуют об этом? Запишите цифры, под которыми они указаны.

1) Экономические пропорции устанавливаются централизованно.

2) Основные средства производства принадлежат государству.

3) Цены на товары определяются соответствием спроса и предложения.

4) Производители сами решают, какое количество товара надо произвести.

5) В стране законодательно гарантируется многообразие форм собственности.

6) Ресурсы распределяются между производителями централизованно.

10. На графике отражена ситуация на рынке вторичного жилья. (На графике P – цена, Q – количество). Проанализируйте изменение спроса (линия спроса D переместилась в положение D1).

Какие из перечисленных факторов могут вызвать такое изменение? Запишите цифры, под которыми они указаны.

1) рост денежных доходов населения

2) сокращение количества строительных фирм

3) активное строительство социального жилья

4) снижение процентов по ипотечным кредитам на приобретение жилья в новостройках

5) серия крупных журналистских расследований о махинациях на рынке вторичного жилья

11. Выберите верные суждения о малой социальной группе и запишите цифры, под которыми они указаны.

1) Эмоциональные контакты не влияют на формирование внутренней структуры межличностных отношений в группе.

2) Малые социальные группы, в отличие от больших групп, всегда носят неформальный характер.

3) Примером малой социальной группы служит население деревни.

4) Одним из отличительных признаков малой социальной группы является наличие постоянной цели совместной деятельности.

5) К групповой культуре относятся нормы, правила, стандарты жизни, поведения, определяющие ожидания членов группы по отношению

друг к другу.

12. Социологические службы страны Z провели опрос. В ходе опроса респондентов спрашивали: «Насколько интересно Вам было бы получать информацию о состоянии окружающей среды?».

Найдите в приведённом списке выводы, которые можно сделать на основе диаграммы, и запишите цифры, под которыми они указаны.

1) Доля респондентов, которые не определились относительно своего интереса к информации о состоянии окружающей среды в месте своего проживания, больше, чем доля тех, кого состояние окружающей среды в месте их проживания не интересует.

2) Экологические проблемы не вызывают существенного интереса у жителей страны Z.

3) Более четверти опрошенных не проявляют интереса к состоянию окружающей среды в своём регионе.

4) Почти две трети опрошенных выражают интерес к состоянию окружающей среды в своём регионе.

5) Доли респондентов, интересующихся состоянием окружающей среды в своей стране в целом, и тех, кому это не интересно, оказались примерно равными.

13. Выберите верные суждения об отличиях молодежи от других социальных групп и запишите цифры, под которыми они указаны.

1) Молодежь отличается от других групп социальной активностью.

2) Перед молодежью стоит задача профессионального самоопределения.

3) В отличие от других социальных групп, жизнедеятельность молодежи целенаправленна.

4) К особенностям социального положения молодежи относится высокий уровень мобильности.

5) Молодежь как социальная группа отличается активным поиском своего места в жизни.

14. Установите соответствие между полномочиями и субъектами государственной власти РФ, реализующими эти полномочия: к каждой позиции, данной в первом столбце, подберите соответствующую позицию из второго столбца.

ПОЛНОМОЧИЯ A) объявление амнистии Б) осуществление помилования B) обеспечение исполнения федерального бюджета Г) принятие федеральных законов Д) осуществление управления федеральной собственностью СУБЪЕКТЫ ГОСУДАРСТВЕННОЙ ВЛАСТИ 1) Президент РФ 2) Государственная Дума 3) Правительство РФ

15. Выберите верные суждения о праве на замену военной службы альтернативной гражданской и запишите цифры, под которыми они указаны.

1) Гражданин имеет право на замену военной службы по призыву альтернативной гражданской службой в случаях, если несение военной службы противоречит его убеждениям или вероисповеданию.

2) На альтернативную службу могут быть направлены граждане, не подлежащие призыву на военную службу.

3) Для направления на гражданскую службу достаточно лично подать заявление в военный комиссариат о желании заменить военную службу по призыву альтернативной гражданской службой.

4) В отношении каждого направляемого на альтернативную гражданскую службу должно быть принято соответствующее решение призывной комиссией.

5) Граждане проходят альтернативную гражданскую службу, как правило, на территории субъектов Российской Федерации, в которых они постоянно проживают.

16. Что из перечисленного относится к группе социально-экономических прав гражданина РФ? Запишите цифры, под которыми они указаны.

1) право на социальное обеспечение по старости 2) право на жизнь 3) право на жилище 4) право на защиту чести и доброго имени 5) право на свободу и личную неприкосновенность

17. Выберите верные суждения о правоохранительных органах РФ и запишите цифры, под которыми они указаны.

1) Правоохранительные органы — это группа государственных органов, которые уполномочены выполнять деятельность по охране законности и правопорядка, а также защите прав и свобод человека.

2) Прокуратура осуществляет уголовное преследование в соответствии со своими полномочиями.

3) Полиция осуществляет надзор за исполнением законов.

4) Нотариусы участвуют в рассмотрении судами уголовных, гражданских, административных дел в качестве государственного обвинителя.

5) Правоохранительные органы РФ выполняют свою задачу посредством применения соответствующих мер согласно закону и соблюдая предусмотренные нормами процедуры.

18. Установите соответствие между возрастом гражданина и правом, которым он может воспользоваться: к каждой позиции, данной в первом столбце, подберите соответствующую позицию из второго столбца.

ПРАВО A) на поиск информации Б) на объединение B) на участие в выборах Президента РФ Г) на участие в демонстрациях Д) на участие в выборах депутатов Государственной Думы РФ ВОЗРАСТ ГРАЖДАНИНА 1) не зависит от возраста 2) совершеннолетний

19. Что входит в систему права? Выберите из приведенного списка нужные позиции и запишите цифры, под которыми они указаны.

1) правовой обычай 2) норма права 3) отрасль права 4) правовая санкция 5) институт права 6) судебный прецедент

20. Прочитайте приведенный ниже текст, в котором пропущен ряд слов. Выберите из предлагаемого списка слова, которые необходимо вставить на место пропусков.

«Особенности характера, специальные способности и уровень общей одаренности влияют на то или иное направление развития жизнедеятельности *** (А) и на его жизнеспособность, работоспособность, трудоспособность. На темпы роста и созревания оказывают влияние образ жизни, способы *** (Б) (игровой, спортивной, учебной), трудовое и общественное поведение, наличие или отсутствие стрессоров, главнейшими из которых являются *** (В) и т.д. Все эти факторы имеют значение как моменты становления *** (Г) субъекта. Понятие «ответственность» выступает важнейшим внутренним *** (Д) его деятельности. Чувство ответственности, долга проявляется в сознательной готовности человека следовать установленным *** (Е), оценивать свои поступки с точки зрения их последствий для окружающих».

Слова в списке даны в именительном падеже. Каждое слово может быть использовано только один раз. Выбирайте последовательно одно слово за другим, мысленно заполняя каждый пропуск. Обратите внимание на то, что слов в списке больше, чем Вам потребуется для заполнения пропусков.

Список терминов:

1) общество 2) человек 3) конфликтные ситуации 4) межличностные отношения 5) деятельность 6) личность 7) регулятор 8) нормы 9) санкции

 

Входной тест по обществознанию 10 класс

Демонстрационный вариант

Аттестационный материал для проведения промежуточной аттестации по обществознанию для обучающихся 9 класса. Демонстрационный вариант КИМ состоит из трех частей, включающих 25 заданий. На его выполнение

Подробнее

Демоверсия ОГЭ 2019 по обществознанию

Демоверсия ОГЭ 2019 по обществознанию Выполнила: Аджароглу Саадет Алааттиновна Часть 1. 1. Какие из перечисленных терминов используется в первую очередь при описании политической сферы общества? 1) племена,

Подробнее

Инструкция по выполнению работы

Экзаменационная работа для проведения государственной итоговой аттестации выпускников IX классов общеобразовательных учреждений (в новой форме) по ОБЩЕСТВОЗНАНИЮ Район Город (населенный пункт) Школа Класс

Подробнее

ЗАДАНИЕ НА САМОСТОЯТЕЛЬНОЕ

ЗАДАНИЕ НА САМОСТОЯТЕЛЬНОЕ ИЗУЧЕНИЕ ПАРАГРАФА Прочитать параграф 14 Делая записи, помни!( «краткость сестра таланта») Фотографируем тетрадь, в которой выполнено задание и отправляем по Netschool или на

Подробнее

Структура ОГЭ по обществознанию

Структура ОГЭ по обществознанию Инструкция по выполнению работы На выполнение экзаменационной работы по обществознанию отводится 3 часа (180 минут). Работа состоит из 2 частей, включающих 31 задание. Минимальный

Подробнее

Планируемые результаты:

Планируемые результаты: — относительно целостное представление об обществе и человеке, о сферах и областях общественной жизни, механизмах и регуляторах деятельности людей; — знание ряда ключевых понятий

Подробнее

3 вариант ОГЭ по обществознанию 2019 год

3 вариант ОГЭ по обществознанию 2019 год Часть 1 Ответом к заданиям 1 20 является одна цифра, которая соответствует номеру правильного ответа. 1. Учёные изучают классовый и национальный состав населения.

Подробнее

Пояснительная записка

Пояснительная записка Назначение проверочной работы контроль состояния уровня сформированности общеучебных и специальных умений и навыков учащихся 8 класса по предмету «Обществознание». Подходы к отбору

Подробнее

ТИПЫ ЭКОНОМИЧЕСКИХ СИСТЕМ

12 ТИПЫ ЭКОНОМИЧЕСКИХ СИСТЕМ Экономическая система это способ организации совместной деятельности людей в обществе. Выбор типа экономической системы зависит от законодательной системы, форм собственности,

Подробнее

Инструкция по выполнению работы

Обществознание. 9 класс. Используется с бланками ответов Вариант 2005-1 Инструкция по выполнению работы На выполнение экзаменационной работы по обществознанию дается 3 часа (180 минут). Работа состоит

Подробнее

«Академика»

Комплексная олимпиада школьников «Академика» [email protected] 1. Необходимо установить соответствия между органами власти в Российской Федерации и ветвями государственной власти: Органы власти 1. Исполнительная

Подробнее

ОБЩЕСТВОЗНАНИЕ В ТАБЛИЦАХ И СХЕМАХ

П. А. Баранов ОБЩЕСТВОЗНАНИЕ В ТАБЛИЦАХ И СХЕМАХ 10 11 классы Москва АСТ 1 УДК 373:3 ББК 60я721 Б24 Баранов, Пётр Анатольевич. Б24 Обществознание в таблицах и схемах: 10 11 классы/ П. А. Баранов. Москва:

Подробнее

Пояснительная записка

Пояснительная записка Данная рабочая программа разработана на основе Федерального компонента Государственного образовательного стандарта основного общего образования и Программы основного общего образования

Подробнее

2.1. Экономика и экономическая наука

83 2.1. Экономика и экономическая наука В последние десятилетия экономической сфере придается исключительно важное значение. Если в начале XX века ключевым образованием для будущих королей считалось военное,

Подробнее

Демонстрационный вариант 2017 года

Обществознание. 8 класс Демонстрационный вариант 2017 г. Обществознание. 8 класс Демонстрационный вариант 2017 г. Демонстрационный вариант 2017 года Инструкция по выполнению работы Экзаменационная работа

Подробнее

Права Человека и гражданина

Права Человека и гражданина 1. Права человека — это: а) исключительная льгота, предоставляемая кому-либо в отличие от других; б) узаконенная возможность что-то делать, удовлетворять свои потребности; в)

Подробнее

Программа экзамена по обществознанию

Программа экзамена по обществознанию Тема 1. Природное и общественное в человеке. Человек как результат биопсихосоциальной эволюции. Потребности и интересы человека. Деятельность человека, ее основные

Подробнее

по ОБЩЕСТВОЗНАНИЮ 9 класс

Итоговая диагностическая работа по ОБЩЕСТВОЗНАНИЮ 9 класс Инструкция по выполнению работы На выполнение работы по обществознанию отводится 45 минут. Работа включает в себя 20 заданий. Контрольная работа

Подробнее

Стартовый мониторинг обществознание 10 класс

РАССМОТРЕННО

на заседании МО__________________

протокол № ____ от ____________2018 г.

руководитель МО _________________

Кирякова Н.В.

СОГЛАСОВАННО

на заседании методического совета

протокол № ___ от ____________ 2018г.

заместитель директора _____________

Сергеева О.И.

Спецификация

Стартового мониторинга для учащихся 10 класса по обществознанию

  1. Назначение КИМ оценить уровень общеобразовательной подготовки по обществознанию учащихся 10 класса. КИМ предназначены для стартового мониторинга

  1. Документы, определяющие содержание КИМ

Содержание стартового мониторинга определяет Федеральный компонент государственного стандарта основного общего образования по обществознанию (приказ Минобразования России от 05.03.2004 № 1089 «Об утверждении федерального компонента государственных стандартов начального общего, основного общего и среднего (полного) общего образования»).

Содержание работ соответствует Федеральному государственному образовательному стандарту основного общего образования (Приказ Минобрнауки РФ от 17 декабря 2010 г. № 1897).

3. Подходы к отбору содержания, разработке структуры КИМ

Объектами контроля выступают дидактические единицы знаний и требования по формированию умений, закрепленные в соответствующих документах (см. пункт 2). Это в первую очередь предметные умения и знания о по предмету.

Задания различаются по форме и уровню трудности, который определяется характером проверяемого знания, уровнем умения и способом познавательной деятельности, необходимым для выполнения задания. Выполнение заданий КИМ предполагает осуществление таких интеллектуальных действий, как распознавание, воспроизведение, извлечение, классификация, систематизация, сравнение, конкретизация, применение знаний (по образцу или в новом контексте) и др. Задания повышенного уровня сложности, в отличие от базовых, предполагают более сложную, как правило, комплексную по своему характеру познавательную деятельность.

4. Характеристика структуры КИМ

Стартовый мониторинг состоит из одной части, задания в которой отличаются по уровню сложности.

Работа содержит задания с выбором одного ответа и выбором нескольких ответов.

Задание считается невыполненным в следующих случаях: а) указан номер неправильного ответа; б) номер ответа не указан.

.

Таблица 1.Распределение заданий по частям работы

Часть

работы

Тип заданий

Число

заданий

Максимальный

первичный балл

Процент от

максимального

первичного балла

1

Часть 1

С выбором ответа

10

10

100%

Итого:

10

10

100%

8. Продолжительность стартового мониторинга

На выполнение стартового мониторинга отводится 15 минут.

9. Система оценивания выполнения отдельных заданий стартового мониторингав целом

Правильно выполненная работа оценивается 10 баллами.

Каждое правильно выполненное задание оценивается 1 баллом.

Шкала пересчёта первичного балла за выполнение стартового мониторинга отметку по пятибалльной шкале

Отметка по пятибалльной шкале

«2»

«3»

«4»

«5»

Общий балл

0 – 4

5 –6

7-8

9-10

Стартовый мониторинг обществознание 10 класс

  1. Способность и возможность отдельного человека или группы общества подчинять своей воле других людей – это

1)социализация 2)престиж 3)власть 4)политика

2. К признакам  правового государства относят

1)наличие парламента 2)разделение властей 3)однопартийность 4)единую идеологию

  1. В государстве Z проводятся массовые внесудебные расправы над оппозицией, насаждается единая идеология. Какой политический режим сложился в государстве Z?

1)парламентский 2)тоталитарный 3)демократический 4)суверенный

  1. Что характеризует политическую партию, в отличие от других объединений граждан?

1)стремление к получению власти 2)объединение единомышленников

3)общие интересы участников 4)удовлетворение потребностей людей

5. Государство Z сформировано несколькими республиками, которые создали общее законодательство и высшие органы власти. Однако республики сохранили часть своей самостоятельности: в них избираются президенты, действуют республиканские законодательные собрания. Какова форма государственного устройства страны Z?

1)республика 2)унитарное государство 3)федеративное государство 4)монархия

6. В государстве Z регулярно проходят свободные конкурентные выборы депутатов парламента. Государство гарантировало права и свободы граждан. Каков политический режим страны Z?

1)авторитарный 2)многопартийный 3)федеративный 4)демократический

7. В стране Z глава государства – князь – получает свою власть по наследству. Граждане избирают парламент, которому принадлежит высшая законодательная власть. Правительство формируется партией, победившей на парламентских выборах. Какова форма правления в стране Z?

1)федеративное государство 2)президентская республика

3)конституционная монархия 4)унитарное государство

8. К отличительным признакам правового государства относится

1)наличие должности уполномоченного по правам человека 2)суверенитет государства

3)деятельность по поддержанию общественного порядка 4)разделение властей

9. Какой пример иллюстрирует деятельность гражданского общества?

1)Парламент принял поправки к закону об общественных объединениях.

2)Жильцы дома провели митинг против строительства гаража на месте спортивного городка.

3)Кандидаты в депутаты начали активную избирательную кампанию.

4)Глава государства провёл встречу с министрами.

10. Страна Z – унитарное государство. Это значит, что

1)в стране Z реализован принцип разделения властей

2)в регионах страны Z могут существовать собственные правительства

3)в регионах страны Z есть собственные конституции

4)страна Z разделена на регионы, которые не имеют политической самостоятельности

1-3

2-2

3-2

4-1

5-3

6-4

7-3

8-4

9-2

10-4

Наши национальные дни | Примечания, видео, контроль качества и тесты | 10 класс> Социальные науки> Наши традиции, социальные ценности и нормы

Определенные дни, в которые вносились большие изменения, события или декларации на благо соответствующего народа, племени, класса или всей страны, называются национальными. В нашем календаре есть определенные дни, которые мы с большим энтузиазмом отмечаем по всей стране. Это дни наших великих достижений, дни, когда мы воздаем дань памяти мученикам или великим лидерам, дни, когда наша история приняла новый поворот.Правительство Непала объявило эти дни Национальными днями. Некоторые национальные дни описаны ниже:

16-й маг в нашей стране отмечается как День мучеников. Это день, когда автократический правитель Рана убил восторженных молодых людей за демократию. В Катманду в течение недели продолжается поминовение мучеников. Люди отдают дань памяти мученикам по всему Непалу. В Катманду министры, правительственные чиновники, политические лидеры и даже простые граждане идут к воротам мучеников в Тундхикхеле и предлагают гирлянды и букеты к бюстам мучеников в честь их вклада в демократию.Они также посещают Текупачали, где был повешен Шукрарадж Шастри, Сифал, где был казнен Дхармабхакта, и Шобхагавати, где были застрелены Дашрат Чанд и Гангалал. Точно так же Сахид Смарак (Мемориал мучеников) в Лайнчауре — это место, где мучеников чествуют, предлагая цветы и т. Д. Точно так же мы также отдаем дань уважения известным мученикам, которые жертвуют своей жизнью во время движения 2062/63 BS, а также движения Мадеш. как войны маоистских народов.

День демократии:

7-й Фалгун — День демократии в Непале.В тот день 2007 года Б.С. Правление Раны было свергнуто, и в стране установилась демократия. Каждый год мы отмечаем День демократии, по этому случаю есть государственный праздник. Люди устраивают шествия в каждом городе, и лидеры обращаются к собраниям, чтобы привлечь внимание к проблеме. Запускаются разные программы.

День защиты детей:

14 сентября 1990 года (29 Бхадра) Непал подписал Международную конвенцию о правах ребенка 1989 года.Мы отмечаем этот день как День защиты детей. В этот день в школах и Бал Мандирах организуются различные программы. Детям раздают сладости, фрукты и одежду. Речи сделаны для того, чтобы люди узнали о правах ребенка.

День конституции:

День конституции — это день объявления конституции страны. После Народного движения II, старая конституция — Конституция Королевства Непал 2047 г. до н.э. была аннулирована, и 1-го марта была обнародована Временная конституция Непала 2063 BS.Таким образом, 1-я Магия — День нашей Конституции, и мы отмечаем этот день ежегодно.

День Локтантры:

Байсакх 11 — день красной буквы в истории Непала. В этот день в 2063 году до н.э. Королю Гьянендре пришлось уступить однодневному Народному движению II. Он должен был вернуть гражданам Непала государственную власть и суверенитет. Этот день в стране отмечается как День Локтранта.

День образования:

День образования раньше отмечался 12 Фалгун, но с 2063 г. по н.э. он отмечается 8 сентября, что является Всемирным днем ​​грамотности.В этот день люди и учреждения, оказавшие ценные услуги делу образования, награждаются и награждаются по специальной программе. Кроме того, медалями и почетными грамотами награждаются кандидаты наук. или с высшими уровнями магистра и бакалавра. Эти школы, чьи результаты SLC являются лучшими в стране или округе, награждаются образовательными щитами.

Международный женский день:

Мы ежегодно отмечаем Международный женский день 8 марта в ознаменование борьбы за равные права с мужчинами.Женщинам, находящимся на государственной службе, предоставляется отпуск. Они организуют программы для повышения осведомленности женщин. Мужчины и женщины равны перед законом, и оба они также должны быть равны в обществе.

AP и Telangana 10-й класс Социальные науки Учебные материалы английского среднего уровня

AP и Telangana 10-го класса Социальные науки Английский средний учебный материал

1. Индия: особенности рельефа

3.Производство и занятость

5. Индийские реки и водные ресурсы

9. Рампур: сельское хозяйство

12. Устойчивое развитие с участием капитала

Часть — II Современный мир и Индия

13. Мир между войнами 1900-1950: Часть — I

14. Мир между войнами 1900-1950: Часть — II

15. Национально-освободительные движения в колониях

16. Национальное движение в Индии — разделение и независимость

17.Создание Конституции независимой Индии

18. Независимая Индия
(Первые 30 лет — 1947-77)

19. Новые политические тенденции 1977–2000 гг.

20. Послевоенный мир и Индия

21. Социальные движения в наше время

22. Граждане и правительства

1. Индия: особенности рельефа

3. Производство и занятость

5. Индийские реки и водные ресурсы

7.Поселение и миграции

8. Рампур: сельское хозяйство

11. Устойчивое развитие с участием капитала

Часть — II Современный мир и Индия

12. Мир между войнами

13. Национально-освободительные движения в колониях

14. Национальное движение в Индии — раздел и независимость

15. Создание Конституции независимой Индии

16. Избирательный процесс в Индии

17.Независимая Индия
(Первые 30 лет — 1947-77)

18. Новые политические тенденции 1977–2000 гг.

19. Послевоенный мир и Индия

20. Социальные движения в наше время

21. Движение за образование штата Телангана


вступительных экзаменов | Бакалавриат

Оценка при поступлении и представленные работы

Большинство абитуриентов должны пройти предметную письменную аттестацию при поступлении либо перед собеседованием, либо на собеседовании.

Они предназначены для дополнения информации, указанной в вашем заявлении, и обеспечения оценки соответствующих навыков и, при необходимости, ваших уровней знаний и понимания для курса, на который подана заявка.

Кроме того, некоторые колледжи просят абитуриентов на некоторые курсы представить примеры их письменных работ — одно или два школьных / университетских сочинения — которые затем могут быть обсуждены на собеседовании.

Подробную информацию об аттестации при поступлении на каждый курс, включая прошлые работы (если применимо), а также о том, требуют ли колледжи обычно представленные работы, можно найти на вкладке «Требования к поступающим» на каждой из страниц курсов.

В таблице ниже указано, есть ли в курсе письменное оценивание перед собеседованием или во время собеседования для поступления на 2021 год.

Оценка перед собеседованием Оценка на собеседовании

Все колледжи требуют предварительной оценки для следующих курсов:

  • Химическая инженерия
  • Информатика
  • Экономика
  • Инженерное дело
  • Английский
  • Земельное хозяйство
  • Медицина
  • Естественные науки
  • Ветеринария

Обратите внимание, что некоторые колледжи могут также потребовать от поступающих пройти письменную оценку, установленную колледжем, на собеседовании в дополнение к оценке перед собеседованием.См. Подробности на страницах отдельных курсов.

Все колледжи требуют прохождения собеседования для следующих курсов:

  • Археология
  • Архитектура
  • Классика
  • История и современные языки
  • История искусства
  • Закон
  • Языкознание
  • Современные и средневековые языки (MML)
  • Философия
  • Богословие, религия и философия религии

В некоторых колледжах может потребоваться оценка на собеседовании для следующих курсов:

  • Англосаксонское, норвежское и кельтское
  • Азиатские и ближневосточные исследования
  • Образование
  • География
  • История
  • История и политика
  • Гуманитарные, социальные и политические науки
  • Математика
  • Музыка
  • Психологические и поведенческие науки

Студенты старше 21 года, поступающие в зрелый колледж

Если вы являетесь зрелым студентом (в возрасте 21 года и старше), подающим заявление в один из зрелых колледжей для прохождения собеседования в Кембридже, вам не требуется проходить какие-либо предварительные оценки перед собеседованием, и вместо этого вы пройдете письменное задание общего формата на собеседовании. .Исключение составляют кандидаты на медицинское образование, которые сдают биомедицинский допуск к экзамену (BMAT), и на ветеринарную медицину, которые проходят оценку перед собеседованием.

Если вы зрелый студент, подающий заявку в зрелый колледж и желаете, чтобы вас рассматривали для прохождения собеседования за границей, или если вы уже зрелый студент, подающий заявку в колледж стандартного возраста, вы должны пройти обязательную предварительную оценку, которую вы необходимо заранее зарегистрироваться.

Как письменная оценка повлияет на заявки?

Экзамены не являются тестами типа «прошел / не сдал» — ваша успеваемость в любом обязательном письменном экзамене не будет рассматриваться изолированно, а будет приниматься во внимание наряду с другими элементами вашего приложения.

Информация для школ / колледжей

Если вы учитель / экзаменатор, ознакомьтесь также с письменной информацией об оценке для школ / колледжей в разделе для учителей на этом веб-сайте.

Дополнительная информация

Если у вас есть какие-либо вопросы о наших письменных оценках, обращайтесь по адресу [email protected].

Требования к поступающим | CBS — Копенгагенская бизнес-школа


ВХОДНЫЕ ТРЕБОВАНИЯ

Вы имеете право, но не , а , гарантированное поступление в CBS, если вы:

  • Выполнить общее вступительное требование о сдаче выпускного экзамена И
  • Выполните особые вступительные требования для программ в вашем приложении

Если вы не соответствуете общим требованиям для поступления на выпускной экзамен второй ступени средней школы, вы можете подать заявление в качестве лица, отпускающего экзамен.Подробнее о подаче заявки в качестве диспенсера и соответствующих критериях читайте далее на этой странице.

Общие требования для поступления

На веб-сайте Министерства высшего образования и науки Дании вы можете узнать, что требуется от вашей страны для выполнения общих требований к выпускным экзаменам в старшей средней школе.

Министерство высшего образования и науки Дании является национальным агентством при Министерстве образования Дании, которое поддерживает интернационализацию образования и обучения в Дании и предоставляет информацию об образовании в Дании.Здесь вы можете увидеть обзор различных стран и оценку уровней образования (включая перевод оценок по датской шкале). Обратите внимание, что это доступно только на датском языке.

Этот раздел предназначен для тех, кто сдал экзамен в старшей средней школе за пределами ЕС, ЕЭЗ и Швейцарии, и для тех, кто сдает определенные экзамены в старшей средней школе ЕС (где мы не можем преобразовать ваш средний балл в датскую систему оценок)

Общая оценка составляется для каждой заявки по следующим критериям:

  • Ваше образование и оценки: мы рассматриваем как ваши оценки в целом, так и оценки по конкретным требованиям для поступления (в контексте шкалы оценок для вашей страны)
  • Соответствующие внеклассные мероприятия: часов за границей, опыт работы, дополнительное образование и т. Д.Подробнее читайте в разделе «Внеучебные мероприятия в квоте 2» выше
  • Мотивационное эссе: вам следует написать мотивационное эссе для каждой программы в вашем приложении. Прочтите «Мотивационное эссе» ниже, чтобы вдохновиться содержанием

Возможно, вам придется перевести ваши документы
Если у вас есть документы, которые составлены не на английском, датском, шведском, норвежском, французском или немецком языках, вы должны загрузить перевод, а также сканирование оригиналов.Документы, написанные не на одном из вышеперечисленных языков, не будут рассматриваться как часть вашего заявления.

Этот раздел предназначен для тех, кто не сдал полный экзамен средней школы.

Ваша заявка будет обработана как получатель, если к вам относится одно из следующих условий:

  • Вы не сдали экзамен в гимназии, но можете подать заявление на основании другого соответствующего опыта и образования.
  • У вас есть программа IBCP (программа международного бакалавриата)
  • Вы сдали экзамен в старшей средней школе, который не эквивалентен датскому экзамену в старшей средней школе, e.грамм. вы получили вальдорфское образование (школа Рудольфа Штайнера).

Заявки на выдачу заявок обрабатываются при выборе Квоты 2 и должны соответствовать правилам и срокам для Квоты 2. Имейте в виду, что очень мало мест в CBS выделено для выдачи заявителей , и процесс отбора может быть очень сложным. По этой причине мы настоятельно рекомендуем всем кандидатам на диспансеризацию загружать мотивационное эссе для всех своих заявок в CBS (независимо от программы).

Вы должны иметь возможность документально подтвердить, что вы прошли как минимум следующее для всех программ бакалавриата, на которые вы хотите подать заявление:

  • Особые требования при поступлении
  • 2-3 дополнительных предмета на уровне гимназии (A, B или C), чтобы в целом вы сдали экзамены как минимум по 6 предметам на уровне гимназии. В общей сложности 6 предметов используются как альтернативное выполнение общих требований для поступления.


Общая оценка проводится для каждого приложения дозирования.Общая оценка включает:

  • Ваше образование: вы должны соответствовать требованиям для поступления на соответствующую программу (программы), как указано в Требованиях для поступления
  • Соответствующие внеклассные мероприятия: периоды за границей, опыт работы, дальнейшее образование и т. Д. См. «Мероприятия в квоте 2» выше.
  • Мотивационное эссе. См. «Мотивационное эссе» выше для вдохновения по содержанию. Вы должны написать мотивационное эссе для каждой программы в вашем приложении.


Возможно, вам придется перевести ваши документы
Если у вас есть документы, которые составлены не на английском, датском, шведском, норвежском, французском или немецком языках, вы должны загрузить перевод, а также сканирование оригиналов. Документы, написанные не на одном из вышеперечисленных языков, не будут рассматриваться как часть вашего заявления.

Особые требования при поступлении

Вы можете увидеть конкретные требования для поступления на интересующую вас программу (-ы) в таблице ниже.Вы должны пройти все определенные вступительные требования.

Вы можете увидеть международные эквиваленты конкретных требований к въезду в наши таблицы сравнения далее по странице.

Если вы подаете заявку на участие в программе, преподаваемой на датском языке, проверьте конкретные требования для поступления в список на датской версии этой страницы.

бакалавриат особые требования при поступлении
Бакалавр делового администрирования и цифрового управления
  • Уровень английского A
  • Математика, уровень B
  • Уровень B по общественным наукам OR Уровень B по международной экономике OR Уровень B по истории идей OR Уровень B по современной истории
Бакалавр международного бизнеса
  • Уровень английского A
  • Математика, уровень B
  • Уровень B по общественным наукам OR Уровень B по международной экономике OR Уровень B по истории идей OR Уровень B по современной истории
Бакалавр международного судоходства и торговли
  • Уровень английского A
  • Математика, уровень B
  • Уровень B по общественным наукам OR Уровень B по международной экономике OR Уровень B по истории идей OR Уровень B по современной истории
Бакалавр международного бизнеса и политики
  • Уровень английского A
  • Математика, уровень B
  • Уровень B по общественным наукам OR Уровень B по международной экономике OR Уровень B по истории идей OR Уровень B по современной истории
Бакалавр делового администрирования и управления услугами
  • Уровень английского A
  • Математика, уровень B
  • Социальные науки, уровень B OR Международная экономика, уровень B OR История идей, уровень B OR Современная история, уровень B
Бакалавр делового администрирования и социологии
  • Уровень английского A
  • Математика, уровень B
  • Уровень B по общественным наукам OR Уровень B по международной экономике OR Уровень B по истории идей OR Уровень B по современной истории

Бакалавриат в области бизнеса, языка и культуры с

  • Уровень английского A
  • Математика, уровень B
  • Уровень B по общественным наукам OR Уровень B по международной экономике OR Уровень B по истории идей OR Уровень B по современной истории
Бакалавр бизнеса, азиатского языка и культуры — Международный бизнес в Азии
  • Уровень английского A
  • Математика, уровень B
  • Уровень B по общественным наукам OR Уровень B по международной экономике OR Уровень B по истории идей OR Уровень B по современной истории

Уровень китайского языка:

  • Кандидаты, принимающие без предварительного знания китайского языка , будут проходить полную 4-летнюю программу (240 баллов ECTS).
  • Кандидаты, получившие оценку китайского А с минимальной оценкой 10,0 , будут автоматически освобождены от курса «Пропедевтика китайского языка 1» (15 баллов ECTS), и в августе с ними свяжутся по поводу проверки уровня языка. Проверочный тест уровня определит, можете ли вы быть освобождены от дальнейших языковых курсов до максимум 60 баллов ECTS. Это будет осуществляться в индивидуальном порядке с учетом индивидуального сценария исследования.
  • Кандидатам, которые подтвердили одну из квалификаций китайского языка, указанную в разделе «Китайский» (внизу этой страницы), в августе свяжутся по поводу проверки уровня владения языком.Тест проверки уровня определяет количество возможных исключений из курсов китайского языка во время программы. В августе свяжутся только с кандидатами, которые загрузили документацию по квалификации китайского языка, утвержденной приемной комиссией, и которым было предложено место учебы.

Уровень японского языка:

  • Кандидаты, принимающие без предварительного знания японского , будут проходить полную 4-летнюю программу (240 баллов ECTS).
  • Кандидаты, принятые с японским A с минимальной оценкой 10.0 будут автоматически освобождены от курса «Пропедевтика китайского языка 1» (15 баллов ECTS), и в августе с ними свяжутся по поводу проверки языкового уровня. Проверочный тест уровня определит, можете ли вы быть освобождены от дальнейших языковых курсов до максимум 60 баллов ECTS. Это будет осуществляться в индивидуальном порядке с учетом индивидуального сценария исследования.
  • Принятые кандидаты, которые подтвердили одну из квалификаций китайского языка, перечисленную в разделе «Китайский» (внизу этой страницы), получат сообщение в августе относительно проверки уровня владения языком.Тест проверки уровня определяет количество возможных исключений из курсов китайского языка во время программы. В августе свяжутся только с кандидатами, которые загрузили документацию по квалификации китайского языка, утвержденной приемной комиссией, и которым было предложено место учебы.

Носители языка

Бакалавр бизнеса, языка и культуры

Выбранный язык не является первым языком, который вы выучили, и не тем языком, который вы чаще всего используете дома.

Возможно, вы изучали его в старших классах средней школы и / или во время обучения по программе бакалавриата, но не получили полного образования на соответствующем языке.

Собственные динамики

Бакалавр бизнеса, языка и культуры

Выбранный язык — это язык, на котором вы выросли и часто говорили дома, когда росли.
Вы можете иметь или не иметь полное формальное образование на соответствующем языке, i.е. полное среднее образование и / или степень бакалавра, преподавание полностью (или почти полностью) на соответствующем языке.
  • Если у вас нет полного формального образования на соответствующем языке, вы не считаются носителем языка в этом контексте. Вы должны документально подтвердить свое владение выбранным иностранным языком, используя одну из трех альтернатив, описанных под заголовками «Альтернативное выполнение …» внизу этой страницы.
  • Если вы двуязычный / многоязычный и выбранный язык является только одним из двух (или более) языков, на которых вы были воспитаны и получили формальное образование, то вы должны дополнить свое заявление личным заявлением, объясняющим ваш точный опыт, и загрузить подтверждающую документацию .Приемная комиссия CBS оценит ваше образование и решит, имеете ли вы право на выбранный региональный языковой курс.
  • Если выбранный язык является первым языком, который вы выучили и на котором получили формальное образование, и вы не владеете каким-либо другим языком (а) на каком-либо другом языке, то вы не соответствуете нашим критериям отбора для соответствующей региональной языковой программы.

Посмотрите, соответствует ли ваша квалификация в наших сравнительных таблицах

Вы можете увидеть наиболее распространенные сравнения для многих стран в наших таблицах ниже.

Список столов:

  • Название второго экзамена среднего образования, необходимого для выполнения общих требований при поступлении
  • сравнительные уровни для различных конкретных вступительных требований
  • минимальная местная оценка, необходимая для выполнения
  • документы, которые вы должны загрузить, чтобы мы могли оценить вашу заявку.

Следующая таблица сравнения действительна только для 2020 года приема.

Проверьте конкретные требования к поступлению для интересующих вас программ (в разделе выше) и сравните их с информацией в таблице ниже, чтобы узнать, выполняете ли вы.

ТРЕБОВАНИЕ

ЭКВИВАЛЕНТ

Выпускной экзамен второй ступени

Reifeprüfung / Matura Reifeprüfung

  • Минимальная общая сумма 11 уроков в неделю в течение 10, 11 и 12 лет (Erste lebende Fremdsprache + wahlpflichtgegenstände на английском языке)
  • Минимальная общая сумма 8 уроков в неделю для 10, 11 и 12 лет.
  • Вступительное требование выполнено автоматически

( Только для бакалавров в области бизнеса, языка и культуры и международного бизнеса в Азии)
  • Французский BA / F-B
  • Испанский A
  • китайский A
  • Минимальная общая сумма 9 уроков в неделю в течение 10, 11 и 12 лет.

Вы считаете себя носителем языка и, следовательно, не имеете права подавать заявку на бакалавриат по бизнесу, языку и культуре с немецким языком.

Дополнительные сведения см. В разделе «Особые требования к поступающим» выше.

  • 2,0 ​​по датской шкале оценок (минимальный проходной балл по одному предмету)
  • Документы, необходимые для оценки вышеуказанного
  • Аттестат зрелости И
  • «Stundentafel» на 10, 11 и 12 годы И
  • Jahreszeugnis, 12.Schulstufe

Расчет среднего балла:
Подсчитывается простой средний балл по всем отдельным австрийским оценкам в экзаменационном сертификате.

Затем этот австрийский средний балл преобразуется в датскую шкалу оценок с помощью приведенной здесь таблицы преобразования (-> Østrig; -> Karakterer. Доступно только на датском языке).


Другие экзамены
Reife- und Diplomprüfung OR Berufsreifeprüfung

Если у вас есть вышеперечисленное, вы будете оценены по Квоте 2: экзамены без конвертируемого среднего балла.Вы всегда должны выполнять определенные вступительные требования. В соответствующих случаях включите обзор часов преподавания по конкретному предмету и учебной программе.

Отбор квалифицированных кандидатов в группу основан на общей оценке, при которой мы смотрим на ваши оценки в целом и с учетом конкретных требований для поступления, мы смотрим на ваши внеклассные занятия и смотрим на ваше мотивационное эссе. Мы не рассчитываем средний балл, и вы не будете оцениваться по квоте 1.

Следующая таблица сравнения действительна только для 2020 года приема.

Проверьте конкретные требования к поступлению для интересующих вас программ (в разделе выше) и сравните их с информацией в таблице ниже, чтобы узнать, выполняете ли вы.

ТРЕБОВАНИЕ

ЭКВИВАЛЕНТ

Выпускной экзамен второй ступени

Certificat d’enseignement Secondaire Supérieur (C.E.S.S.): Section générale OR техника enseignement de transition OR artistique enseignement de transition

  • Ни один экзамен не соответствует требованиям, см. «Альтернативные способы выполнить требование» ниже
  • Обязательный курс математики соответствует требованию
  • Обязательный курс по истории и географии соответствует требованию

( Только для бакалавров в области бизнеса, языка и культуры и международного бизнеса в Азии)
  • Немецкий BA / F-B
  • Испанский A
  • китайский A
  • Минимум 4 урока в неделю для 10, 11 и 12 лет

Вы считаете себя носителем языка и, следовательно, не имеете права подавать заявку на получение степени бакалавра по бизнесу, языку и культуре с французским языком.

Дополнительные сведения см. В разделе «Особые требования к поступающим» выше.

  • 2,0 ​​по датской шкале оценок (минимальный проходной балл по одному предмету)
  • Документы, необходимые для оценки вышеуказанного
  • Certificat d’enseignement Secondaire Supérieur (C.E.S.S.): Section générale OR , техника OR artistique AND
  • Табель успеваемости, включая оценки и часы за последние 3 года

Расчет среднего балла:
Рассчитывается простой средний балл по всем отдельным бельгийским оценкам в экзаменационном сертификате, за исключением курса поведения: «канал поведения».

Затем этот бельгийский средний балл преобразуется в датскую шкалу оценок с помощью приведенной здесь таблицы преобразования (-> Belgien — det franske fælleskab; -> Karakterer. Доступно только на датском языке).

Другие экзамены

  • Certificat d’enseignement Secondaire Supérieur (C.E.S.S.): квалификация техники раздела
  • Certificat d’enseignement Secondaire Supérieur (C.E.S.S.): Section artistique enseignement de qualification
  • Certificat d’enseignement Secondaire Supérieur (C.E.S.S.): Раздел Professionalelle

Если у вас есть вышеперечисленное, вы будете оценены по Квоте 2: экзамены без конвертируемого среднего балла. Вы всегда должны выполнять определенные вступительные требования. В соответствующих случаях включите обзор часов преподавания по конкретному предмету и учебной программе.

Отбор квалифицированных кандидатов в группу основан на общей оценке, при которой мы смотрим на ваши оценки в целом и с учетом конкретных требований для поступления, мы смотрим на ваши внеклассные занятия и смотрим на ваше мотивационное эссе.Мы не рассчитываем средний балл, и вы не будете оцениваться по квоте 1.

Следующая таблица сравнения действительна только для 2020 года приема.

Проверьте конкретные требования к поступлению для интересующих вас программ (в разделе выше) и сравните их с информацией в таблице ниже, чтобы узнать, выполняете ли вы.

ТРЕБОВАНИЕ

ЭКВИВАЛЕНТ

Выпускной экзамен второй ступени

Диплом за (Заваршено) Среднее образование / за Среднее образование

  • Английский как первый иностранный язык (556 часов) И интенсивное изучение английского языка (648 часов)
  • Минимальная общая сумма 335 уроков в 10, 11 и 12 классы
  • Минимальная общая сумма 200 уроков (по истории и / или обществознанию) в 10, 11 и 12 классы

( Только для бакалавров в области бизнеса, языка и культуры и международного бизнеса в Азии)
  • Французский BA / F-B
  • Немецкий BA / F-B
  • Испанский A
  • китайский A
  • Минимальная общая сумма 435 уроков в 10, 11 и 12 классах
  • 2.0 по датской шкале оценок (минимальный проходной балл по отдельному предмету)
  • Документы, необходимые для оценки вышеуказанного
  • Диплом за (Заваршено) Среднее образование / за Среднее образование И
  • Апостиль И
  • Табель успеваемости за 10, 11 и 12 классы

Расчет GPA:

Болгарский средний балл, указанный в экзаменационном сертификате, конвертируется в датскую шкалу оценок с использованием приведенной здесь таблицы преобразования (Bulgarien; -> Karakterer.Доступно только на датском языке).

Другие экзамены

  • Диплом за (Заваршено) Среднее образование / за Среднее образование — по специальности «Бизнес»
  • Диплом за Среднее специальное образование / за Среднее образование — техническое или бизнес

Если у вас есть вышеперечисленное, вы будете оценены по Квоте 2: экзамены без конвертируемого среднего балла. Вы всегда должны выполнять определенные вступительные требования. В соответствующих случаях включите обзор часов преподавания по конкретному предмету и учебной программе.

Отбор квалифицированных кандидатов в группу основан на общей оценке, при которой мы смотрим на ваши оценки в целом и с учетом конкретных требований для поступления, мы смотрим на ваши внеклассные занятия и смотрим на ваше мотивационное эссе. Мы не рассчитываем средний балл, и вы не будете оцениваться в Квоте 1

.

Этот раскрывающийся список не работает

Следующая таблица сравнения действительна только для 2020 года приема.

Проверьте конкретные требования к поступлению для интересующих вас программ (в разделе выше) и сравните их с информацией в таблице ниже, чтобы узнать, выполняете ли вы.

ТРЕБОВАНИЕ

ЭКВИВАЛЕНТ

Выпускной экзамен второй ступени

Vysvedcení o Maturitni zkoušce

  • Ни один экзамен не соответствует требованиям, см. «Альтернативные способы выполнить требование» ниже
  • Минимум 3.5 уроков в неделю во 2, 3 и 4 классе Gymnázium
  • Минимум 2 урока (по истории и / или обществознанию) в неделю на 2, 3 и 4 классах гимназии

( только для бакалавров в области бизнеса, языка и культуры и международного бизнеса в Азии)

  • Французский BA / F-B
  • Немецкий BA / F-B
  • Испанский A
  • китайский A
  • Минимум 4.5 уроков в неделю во 2, 3 и 4 классе Gymnázium
  • 2,0 ​​по датской шкале оценок (минимальный проходной балл по одному предмету)
  • Документы, необходимые для оценки вышеуказанного
  • Vysvedcení o Maturitni zkoušce AND
  • Результаты экзаменов за все годы Vysvedcení o Maturitni zkoušce AND
  • Школьный документ с указанием количества уроков в неделю за последние 3 года

Расчет среднего балла:
Средний балл по всем оценкам экзаменационного сертификата ( Vysvedcení o Maturitni zkoušce ) вычисляется с округлением до одного десятичного знака.

Средний балл всех оценок за последний семестр «Высведчены» рассчитывается с округлением до одного десятичного знака.

Среднее значение этих двух средних баллов затем вычисляется и конвертируется в датскую шкалу оценок с использованием приведенной здесь таблицы преобразования ( Доступно только на датском языке ).

Другие экзамены

  • Vysvedcení o Maturitni zkoušce / Maturitní zkouška / Maturita from Srední odborná skola
  • Maturitní zkouška от Konzervator
  • Maturitní zkouška из Strední odborné uciliste

Если у вас есть указанное выше, вы будете оценены как дозатор в Квоте 2.Диспенсер всегда должен соответствовать определенным требованиям для въезда. В соответствующих случаях включите обзор часов преподавания по конкретному предмету и учебной программе.

Выбор квалифицированных медработников основан на общей оценке, при которой мы смотрим на ваши оценки в целом и в отношении конкретных требований для поступления, мы смотрим на ваши внеклассные мероприятия и смотрим на ваше мотивационное эссе. Мы не рассчитываем средний балл, и вы не будете оцениваться по квоте 1.

Следующая таблица сравнения действительна только для 2020 года приема.

Проверьте конкретные требования к поступлению для интересующих вас программ (в разделе выше) и сравните их с информацией в таблице ниже, чтобы узнать, выполняете ли вы.

ТРЕБОВАНИЕ

ЭКВИВАЛЕНТ

Выпускной экзамен второй ступени

  • Maturalna svjedodžba OR
  • Свъедодзба о заврсеном среднем образовании ИЛИ
  • Свъедоджба о заврсном испиту-матури ИЛИ
  • Сведоджба о Државной Матури
  • Ни один экзамен не соответствует требованиям, см. «Альтернативные способы выполнить требование» ниже
  • Минимальная общая сумма 10 уроков в неделю в течение 10, 11 и 12 лет
  • Минимальная общая сумма 6 занятий в неделю в течение 10, 11 и 12 лет

( только для бакалавров в области бизнеса, языка и культуры и международного бизнеса в Азии)

  • Французский BA / F-B
  • Немецкий BA / F-B
  • Испанский A
  • китайский A
  • Ни один экзамен не соответствует требованиям, см. «Альтернативные способы выполнить требование» ниже
  • 2.0 по датской шкале оценок (минимальный проходной балл по отдельному предмету)
  • Документы, необходимые для оценки вышеуказанного
  • Сведоджба о Државной Матури И
  • Итоговые результаты экзаменов за 4 года

Расчет GPA:

Средний балл всех оценок на государственном экзамене «Свъедоджба о Државной Матури» вычисляется с округлением до одного десятичного знака.

Средний академический балл всех оценок за последний год «Свъедоджба» вычисляется с округлением до одного десятичного знака.

Среднее значение этих двух средних баллов затем вычисляется и конвертируется в датскую шкалу оценок с использованием приведенной здесь таблицы преобразования ( Доступно только на датском языке ).

Другие экзамены

  • Свъедоджба о завршном испиту
  • Свъедодзба о заврсеном среднем образовании
  • Свъедодзба о заврсном испиту-матури
  • Свъедозба о матурь

Если у вас есть один из вышеперечисленных, вы будете оценены как дозатор в Квоте 2.Диспенсер всегда должен соответствовать определенным требованиям для въезда. В соответствующих случаях включите обзор часов преподавания по конкретному предмету и учебной программе.

Выбор квалифицированных медработников основан на общей оценке, при которой мы смотрим на ваши оценки в целом и в отношении конкретных требований для поступления, мы смотрим на ваши внеклассные мероприятия и смотрим на ваше мотивационное эссе. Мы не рассчитываем средний балл, и вы не будете оцениваться по квоте 1.

Следующая таблица сравнения действительна только для 2020 года приема.

Проверьте конкретные требования к поступлению для интересующих вас программ (в разделе выше) и сравните их с информацией в таблице ниже, чтобы узнать, выполняете ли вы.

ТРЕБОВАНИЕ

ЭКВИВАЛЕНТ

Выпускной экзамен второй ступени

Данско-Французский бакалавриат (DFB)

  • Обязательный курс датского языка A соответствует требованию
  • Ни один экзамен не соответствует требованиям, см. «Альтернативные способы выполнить требование» ниже
  • Обязательный курс английского языка соответствует требованию
  • Обязательный курс математики соответствует требованию
  • Обязательный курс истории соответствует требованию
( Только для бакалавров в области бизнеса, языка и культуры и международного бизнеса в Азии)
  • Немецкий BA / F-B
  • Испанский A
  • китайский A
  • Минимальная общая сумма 10 уроков в неделю во время Dansk-Fransk Baccalauréat
Вы являетесь носителем языка и, следовательно, не имеете права подавать заявку на получение степени бакалавра по бизнесу, языку и культуре с французским языком.

Дополнительные сведения см. В разделе «Особые требования к поступающим» выше.

  • 2,0 ​​по датской шкале оценок (минимальный проходной балл по одному предмету)
  • Документы, необходимые для оценки вышеуказанного
  • Экзаменационный сертификат И
  • Релевантные заметки

Расчет среднего балла:
Как преобразовать средний балл и таблицу преобразования между французской и датской шкалами оценок можно найти здесь (Frankrig; -> Karakterer.Доступно только на датском языке).

Следующая таблица сравнения действительна только для 2020 года приема.

Проверьте конкретные требования к поступлению для интересующих вас программ (в разделе выше) и сравните их с информацией в таблице ниже, чтобы узнать, выполняете ли вы.

ТРЕБОВАНИЕ

ЭКВИВАЛЕНТ

Выпускной экзамен второй ступени

Европейский бакалавриат

  • L III — 4 года
  • L IV — 4 года (4 часа)
  • Обязательный курс математики соответствует требованию
  • Обязательный курс истории соответствует требованию
( Только для бакалавров в области бизнеса, языка и культуры и международного бизнеса в Азии)
  • Французский BA / F-B
  • Немецкий BA / F-B
  • Испанский A
  • китайский A
  • LI
  • ЛИИ
  • LIII
  • LIV, если проучился 4 года (необходимо загрузить документацию за все 4 года)
  • 2.0 по датской шкале оценок (минимальный проходной балл по отдельному предмету)
  • Документы, необходимые для оценки вышеуказанного
  • Диплом И
  • Оценка успеваемости


Расчет среднего балла:
Средний балл, указанный в дипломе, конвертируется в датскую шкалу оценок с использованием приведенной здесь таблицы преобразования (European Baccalaureate / Europæisk Studentereksamen; -> Karakterer.Доступно только на датском языке).

Требования к Европейскому бакалавриату / Europæisk Studentereksamen:

European Baccaluareate / Europæisk Studentereksamen должен соответствовать всем из следующих критериев, чтобы соответствовать общим требованиям для поступления:

  • Средний балл должен быть не менее 60%
  • До 2013 года включительно: минимум 2 из 4 устных экзаменов должны быть языковыми курсами, включая родной язык (язык 1 / L1), и минимум 2 из 5 письменных экзаменов должны быть на одном из языков курсы, включая родной язык (язык 1 / L1)
  • По состоянию на 2014 год включительно: 1 из 3 устных экзаменов должен быть родным языком (или языком преподавания языковой секции, в которой учится), а 3 из 5 письменных экзаменов должны быть родным языком, первым иностранным языком и Математика

Следующая таблица сравнения действительна только для 2020 года приема.

Проверьте конкретные требования к поступлению для интересующих вас программ (в разделе выше) и сравните их с информацией в таблице ниже, чтобы узнать, выполняете ли вы.

ТРЕБОВАНИЕ

ЭКВИВАЛЕНТ

Выпускной экзамен второй ступени

Keskkooli Loputunnistus / Gümnaasiumi Loputunnistus & Riigieksamenitunnistus

  • Ни один экзамен не соответствует требованиям, см. «Альтернативные способы выполнить требование» ниже
  • Мин.9 «kursuste arv» по математике
  • Мин. 6 «kursuste arv» по истории и / или обществознанию
( Только для бакалавров в области бизнеса, языка и культуры и международного бизнеса в Азии)
  • Французский BA / F-B
  • Немецкий BA / F-B
  • Испанский A
  • китайский A
  • Мин. 12 «kursuste arv» на соответствующем языке
  • 2.0 по датской шкале оценок (минимальный проходной балл по отдельному предмету)
  • 3 (Loputunnistus)
  • 20-28 (Riigieksamenitunnistus)
  • Документы, необходимые для оценки вышеуказанного
  • Gümnaasiumi Lõputunnistus AND
  • Hinnetekeht AND
  • Riigieksamenitunnistus

Расчет среднего балла:
Средний балл государственного сертификата ( Riigieksamenitunnistus ) рассчитывается и конвертируется в датскую шкалу оценок (см. Ниже *).

Средний балл аттестата школьного экзамена ( Gümnaasiumi Loputunnistus ) рассчитывается и конвертируется в датскую шкалу оценок (см. Ниже *).

Затем вычисляется среднее значение этих двух GPA.

Таблицу преобразования между эстонскими и датскими оценками можно найти здесь (Эстония; -> Карактерер. Доступно только на датском языке).


Другие экзамены

  • Среднее образование, совмещенное с бизнес-курсами

Если у вас есть вышеперечисленное, вы будете оценены по Квоте 2: экзамены без конвертируемого среднего балла.Вы всегда должны выполнять определенные вступительные требования. В соответствующих случаях включите обзор часов преподавания по конкретному предмету и учебной программе.

Отбор квалифицированных кандидатов в группу основан на общей оценке, при которой мы смотрим на ваши оценки в целом и с учетом конкретных требований для поступления, мы смотрим на ваши внеклассные занятия и смотрим на ваше мотивационное эссе. Мы не рассчитываем средний балл, и вы не будете оцениваться по квоте 1.

Следующая таблица сравнения действительна только для 2020 года приема.

Проверьте конкретные требования к поступлению для интересующих вас программ (в разделе выше) и сравните их с информацией в таблице ниже, чтобы узнать, выполняете ли вы.

ТРЕБОВАНИЕ

ЭКВИВАЛЕНТ

Выпускной экзамен второй ступени

Ylioppilastutkintotodistus / Studentexamensbetyg

(Только для датских программ erhvervsøkonomi (HA))

  • Свенск как родной язык 8 курсер
  • Svensk as A-språk
  • Svensk as B-språk 6 kurser

  • Ни один экзамен не соответствует требованиям, см. «Альтернативные способы выполнить требование» ниже
  • Математика мин.13 курсер
  • Математика мин. 8 курсер
  • История и / или обществознание мин. 6 курсер
( Только для бакалавров в области бизнеса, языка и культуры и международного бизнеса в Азии)
  • Французский BA / F-B
  • Немецкий BA / F-B
  • Испанский A
  • китайский A
  • Уровень: мин.8 курсеров на соответствующем языке
    Другое: мин. 11 курсер на соответствующем языке
  • 2,0 ​​по датской шкале оценок (минимальный проходной балл по одному предмету)
  • Документы, необходимые для оценки вышеуказанного
  • Ylioppilastutkintotodistus / Studentexamensbetyg И
  • Lukion päästötodistus / Angångsbetyg from Gymnasiet

Расчет среднего балла:
Рассчитайте A) средний балл по Lukion päättötodistus / Avgångsbetyg и B) средний балл по Ylioppilastutkintotodistus / Studentereksamensbeviset

А.Lukion päättötodistus / Avgångsbetyg: средний балл рассчитывается как простой средний балл оценок, указанных в сертификате.

B. Ylioppilastutkintotodistus / Studentereksamensbeviset:

1: Сначала преобразуйте результаты экзаменов (например, Approbatur и laudatur) для отдельных курсов в числа

.

2: Сложите оценки и вычислите средний балл. Включите как 4 обязательных экзамена, так и экзамены по выбору.

2) Сложите средний балл от A и средний балл от B вместе и разделите на 2.Преобразуйте это в датский средний балл по 7-ступенчатой ​​шкале, используя приведенную ниже таблицу преобразования.

Таблицы преобразования между финскими и датскими оценками можно найти здесь (Финляндия; -> Karakterer. Доступно только на датском языке).

Другие экзамены

  • Tutkintotodistus / Ammatillinen perustutkinto
  • Tutkinto 3 vuotta lyhyemmistä ammattiopinnoista
  • Tutkintotodistus / Näyttötutkintona suoritettu ammatillinen perustutkinto
  • Tutkintotodistus / Ammattitutkinto

Если у вас есть вышеперечисленное, вы будете оценены по Квоте 2: экзамены без конвертируемого среднего балла.Вы всегда должны выполнять определенные вступительные требования. В соответствующих случаях включите обзор часов преподавания по конкретному предмету и учебной программе.

Отбор квалифицированных кандидатов в группу основан на общей оценке, при которой мы смотрим на ваши оценки в целом и с учетом конкретных требований для поступления, мы смотрим на ваши внеклассные занятия и смотрим на ваше мотивационное эссе. Мы не рассчитываем средний балл, и вы не будете оцениваться по квоте 1.

Следующая таблица сравнения действительна только для 2020 года приема.

Проверьте конкретные требования к поступлению для интересующих вас программ (в разделе выше) и сравните их с информацией в таблице ниже, чтобы узнать, выполняете ли вы.

ТРЕБОВАНИЕ

ЭКВИВАЛЕНТ

Выпускной экзамен второй ступени

Baccalauréat général

  • Ни один экзамен не соответствует требованиям, см. «Альтернативные способы выполнить требование» ниже
  • Обязательный курс математики соответствует требованию
  • Обязательный курс истории-географии соответствует требованию
( Только для бакалавров в области бизнеса, языка и культуры и международного бизнеса в Азии)
  • Немецкий BA / F-B
  • Испанский A
  • китайский A
  • Минимальная общая сумма 10 еженедельных уроков соответствующего языка во время Baccalauréat général

Вы считаете себя носителем языка и, следовательно, не имеете права подавать заявку на получение степени бакалавра по бизнесу, языку и культуре с французским языком.

Дополнительные сведения см. В разделе «Особые требования к поступающим» выше.

  • 2,0 ​​по датской шкале оценок (минимальный проходной балл по одному предмету)
  • Документы, необходимые для оценки вышеуказанного
  • Общий диплом бакалавра И
  • Релевантные заметки
  • Карнет нот


Расчет среднего балла:
Французский средний балл, указанный в примечаниях к бакалавриату, конвертируется в датскую шкалу оценок.Эту таблицу преобразования можно найти здесь (Frankrig; -> Karakterer. Доступно только на датском языке).

Другие экзамены

  • Технология бакалавриата
  • Baccalauréat Professionalnel
  • Diplôme d’accès aux ètudes Universitaires (D.A.E.U.)

Если у вас есть вышеперечисленное, вы будете оценены по Квоте 2: экзамены без конвертируемого среднего балла. Вы всегда должны выполнять определенные вступительные требования.В соответствующих случаях включите обзор часов преподавания по конкретному предмету и учебной программе.

Отбор квалифицированных кандидатов в группу основан на общей оценке, при которой мы смотрим на ваши оценки в целом и с учетом конкретных требований для поступления, мы смотрим на ваши внеклассные занятия и смотрим на ваше мотивационное эссе. Мы не рассчитываем средний балл, и вы не будете оцениваться по квоте 1.

Следующая таблица сравнения действительна только для 2020 года приема.

Проверьте конкретные требования к поступлению для интересующих вас программ (в разделе выше) и сравните их с информацией в таблице ниже, чтобы узнать, выполняете ли вы.

ТРЕБОВАНИЕ

ЭКВИВАЛЕНТ

Выпускной экзамен второй ступени

  • Уровни GCSE, AS- и A (минимум 9 различных предметов, включая минимум 1 полный уровень A и комбинацию, эквивалентную 3 уровням A)
  • Кандидаты только с уровнями AS и A (если эквивалент GCSE был взят в другой образовательной системе): минимум 2 предмета на уровне AS и 3 предмета на уровне A (всего 5 разных предметов).Один из уровней AS может быть заменен на уровень 3 «Расширенный проект»
  • Уровень английского языка
  • A level English Литература
  • Уровень английского языка GSCE и документы для начальной и средней школы английского языка, принятые в Великобритании
  • История на уровне GCSE
  • Социология на уровне GCSE
  • Экономика на уровне GCSE
  • Бизнес-исследования на уровне GCSE
( Только для бакалавров в области бизнеса, языка и культуры и международного бизнеса в Азии)
  • Французский BA / F-B
  • Немецкий BA / F-B
  • Испанский A
  • китайский A
  • Уровень соответствующего языка
  • 2.0 по датской шкале оценок (минимальный проходной балл по отдельному предмету)
  • GCSE: G или 1
  • Уровни AS- / A: E
  • Документы, необходимые для оценки вышеуказанного
  • Все окончательные сертификаты GCSE, сертификаты уровня AS и сертификаты уровня A NB , Мы не можем использовать прогнозируемые оценки или Заявление о предварительных результатах кандидата в качестве окончательной документации.Если вы потеряли свои сертификаты GCSE или A level, вы можете заказать Сертифицированный отчет о результатах в соответствующей экзаменационной комиссии. Для получения дополнительной информации см. Здесь.

Расчет среднего балла:
Все результаты уровней GCSE, AS- и A конвертируются в число по датской системе оценок (которое достигает 12). Любые экзамены уровня AS, сданные также на уровне A, исключаются из уравнения. Числа взвешены (GCSE = 1; AS level = 1.5 и уровень A = 2). Например, A * на уровне GCSE = 12 взвешивается один раз, тогда как A * на уровне A = 12 взвешивается дважды (поэтому 24 делится на 2 при вычислении среднего), а затем мы вычисляем (взвешенное) среднее.

Числовое преобразование оценок и пример вышеперечисленного можно найти здесь ( Доступно только на датском языке ).

Другие экзамены

  • Cambridge Pre-U
  • BTEC — Национальный сертификат / Национальный диплом
  • Диплом о доступе к высшему образованию

Если у вас есть указанное выше, вы будете оценены как дозатор в Квоте 2.Диспенсер всегда должен соответствовать определенным требованиям для въезда. В соответствующих случаях включите обзор часов преподавания по конкретному предмету и учебной программе.

Выбор квалифицированных медработников основан на общей оценке, при которой мы смотрим на ваши оценки в целом и в отношении конкретных требований для поступления, мы смотрим на ваши внеклассные мероприятия и смотрим на ваше мотивационное эссе. Мы не рассчитываем средний балл, и вы не будете оцениваться по квоте 1.

Следующая таблица сравнения действительна только для 2020 года приема.

Проверьте конкретные вступительные требования для интересующих вас программ (в разделе выше) и сравните их с информацией в таблице ниже, чтобы узнать, выполняете ли вы.

ТРЕБОВАНИЕ

ЭКВИВАЛЕНТ

  • Выпускной экзамен второй ступени средней школы
  • Уровни iGCSE, iAS- и iA (минимум 9 различных предметов, включая минимум 1 полный уровень iA и комбинацию, эквивалентную 3 уровням A)
  • Кандидаты только с уровнями AS и A (если эквивалент GCSE был взят в другой образовательной системе): минимум 2 предмета на уровне AS и 3 предмета на уровне A (всего 5 разных предметов).Один из уровней AS может быть заменен на уровень 3 «Расширенный проект»
  • Уровень английского языка
  • A level English Литература
  • История на уровне GCSE
  • Социология на уровне GCSE
  • Экономика на уровне GCSE
  • Бизнес-исследования на уровне GCSE
( Только для бакалавров в области бизнеса, языка и культуры и международного бизнеса в Азии)
  • Французский BA / F-B
  • Немецкий BA / F-B
  • Испанский A
  • китайский A
  • Уровень соответствующего языка
  • 2.0 по датской шкале оценок (минимальный проходной балл по отдельному предмету)
  • GCSE: G или 1
  • Уровни AS- / A: E
  • Документы, необходимые для оценки вышеуказанного
  • Все окончательные сертификаты iGCSE, сертификаты уровня iAS и сертификаты уровня iA NB , Мы не можем использовать прогнозируемые оценки или Отчет о результатах в качестве окончательной документации

Расчет среднего балла:
Все результаты уровней GCSE, AS- и A конвертируются в число по датской системе оценок (которое достигает 12).Любые экзамены уровня AS, сданные также на уровне A, исключаются из уравнения. Числа взвешены (GCSE = 1; уровень AS = 1,5 и уровень A = 2). Например, A * на уровне GCSE = 12 взвешивается один раз, тогда как A * на уровне A = 12 взвешивается дважды (поэтому 24 делится на 2 при вычислении среднее), а затем вычисляем (взвешенное) среднее.

Числовое преобразование оценок и пример вышеперечисленного можно найти здесь ( Доступно только на датском языке ).

Следующая таблица сравнения действительна только для 2020 года приема.

Проверьте конкретные требования к поступлению для интересующих вас программ (в разделе выше) и сравните их с информацией в таблице ниже, чтобы узнать, выполняете ли вы.

ТРЕБОВАНИЕ

ЭКВИВАЛЕНТ

Выпускной экзамен второй ступени

Zeugnis der Allgemeinen Hochschulreife (Abiturzeugnis)

Английский язык с «erhöhtes Anforderungsniveau» как часть Abitur., например:

  • Английский как «Leistungskurse» — должен быть отмечен буквой L или LK в «Zeugnis der Allgemeinen Hochschulreife»
  • Английский как «Profilfach» — должен быть отмечен буквой P в «Zeugnis der Allgemeinen Hochschulreife»
  • Английский как «Neigungsfach» — должен быть отмечен буквой N в «Zeugnis der Allgemeinen Hochschulreife»
  • Английский как «Kernfach» — должен быть «Prüfungsfach» с письменным экзаменом на «Leistungen in der Abiturprüfung»
  • Английский язык отмечен знаком eA в «Zeugnis der Allgemeinen Hochschulreife»
  • Английский язык отмечен двойкой в ​​»Zeugnis der Allgemeinen Hochschulreife»
  • Математика как Grundkurs в течение 2 лет Abitur
  • История во время Abitur
  • Общественные науки во время Abitur
( Только для бакалавров в области бизнеса, языка и культуры и международного бизнеса в Азии)
  • Французский BA / F-B
  • Испанский A
  • китайский A

Соответствующий язык сдан с «erhöhtes Anforderungsniveau» как часть Abitur, например:

  • «Leistungskurs»
  • «Профильфач»
  • ‘Neigungsfach’
  • «Kernfach»
Немецкий BA / F-B

Вы считаете себя носителем языка и, следовательно, не имеете права подавать заявку на бакалавриат по бизнесу, языку и культуре с немецким языком.

Дополнительные сведения см. В разделе «Особые требования к поступающим» выше.

  • 2,0 ​​по датской шкале оценок (минимальный проходной балл по одному предмету)
  • 4 (5 пунктов)
  • 4 (6 пунктов)
  • Документы, необходимые для оценки вышеуказанного
  • Экзаменационный сертификат Abiturzeugnis AND
  • Результаты экзаменов (выпускных и полугодовых) за 2 выпускных года

Расчет среднего балла:
Немецкий средний балл ( Durchschnittsnote ), указанный в Abiturzeugnis, преобразован в датскую шкалу оценок.Эту таблицу преобразования можно найти здесь (Tyskland; -> Karakterer. Доступно только на датском языке).

Другие экзамены

  • Zeugnis der Fachgebundenen Hochschulreife
  • Fachhochschulreife


Если у вас есть вышеперечисленное, вы будете оценены по Квоте 2: экзамены без конвертируемого среднего балла. Вы всегда должны выполнять определенные вступительные требования. В соответствующих случаях включите обзор часов преподавания по конкретному предмету и учебной программе.

Отбор квалифицированных кандидатов в группу основан на общей оценке, при которой мы смотрим на ваши оценки в целом, а в отношении конкретных требований для поступления, мы смотрим на ваши внеклассные занятия и смотрим ваше мотивационное эссе. Мы не рассчитываем средний балл, и вы не будете облагаться квотой 1.

Следующая таблица сравнения действительна только для 2020 года приема.

Проверьте конкретные требования к поступлению для интересующих вас программ (в разделе выше) и сравните их с информацией в таблице ниже, чтобы узнать, выполняете ли вы.

ТРЕБОВАНИЕ

ЭКВИВАЛЕНТ

Выпускной экзамен второй ступени

Apolitírio Enaíou Lykeíou / Apolytirion Geniko Lykeiou

  • Ни один экзамен не соответствует требованиям, см. «Альтернативные способы выполнить требование» ниже
  • Обязательный курс математики соответствует требованию
  • Обязательный курс истории соответствует требованию
( Только для бакалавров в области бизнеса, языка и культуры и международного бизнеса в Азии)
  • Французский BA / F-B
  • Немецкий BA / F-B
  • Испанский A
  • китайский A
  • Ни один экзамен не соответствует требованиям, см. «Альтернативные способы выполнить требование» ниже
  • 2.0 по датской шкале оценок (минимальный проходной балл по отдельному предмету)
  • По состоянию на 2000 год : 9,5 — 10,5
  • До 2000 г. : 10,0 — 11,0
  • Документы, необходимые для оценки вышеуказанного
  • Apolitírio Enaíou Lykeíou / Apolytirion Geniko Lykeiou И
  • Vevaiosi Prosvasis

Расчет среднего балла:
Среднее значение среднего балла балла Apolitírio Enaíou Lykeíou и среднего балла балла Vevaiosi Prosvasis преобразовано в датскую шкалу оценок.Эту таблицу преобразования можно найти здесь (Grækenland; -> Karakterer. Доступно только на датском языке).

Другие экзамены

  • Apolitírio Enaíou Lykeíou / Apolytirion Geniko Lykeiou без Vevaiosi Prosvasis
  • Pthychíon Techniká kai epagelmatiká ekpedeftíria (TEE)
  • Аполитирио Эпангельматико Ликио

Если у вас есть вышеперечисленное, вы будете оценены по Квоте 2: экзамены без конвертируемого среднего балла.Вы всегда должны выполнять определенные вступительные требования. В соответствующих случаях включите обзор часов преподавания по конкретному предмету и учебной программе.

Отбор квалифицированных кандидатов в группу основан на общей оценке, при которой мы смотрим на ваши оценки в целом и с учетом конкретных требований для поступления, мы смотрим на ваши внеклассные занятия и смотрим на ваше мотивационное эссе. Мы не рассчитываем средний балл, и вы не будете оцениваться по квоте 1.

Следующая таблица сравнения действительна только для 2020 года приема.

Проверьте конкретные требования к поступлению для интересующих вас программ (в разделе выше) и сравните их с информацией в таблице ниже, чтобы узнать, выполняете ли вы.

ТРЕБОВАНИЕ

ЭКВИВАЛЕНТ

Выпускной экзамен второй ступени

Erettsigi vizsga / Matura

  • Ни один экзамен не соответствует требованиям, см. «Альтернативные способы выполнить требование» ниже
  • Обязательный курс математики соответствует требованию
  • Обязательный курс истории соответствует требованию
( Только для бакалавров в области бизнеса, языка и культуры и международного бизнеса в Азии)
  • Французский BA / F-B
  • Немецкий BA / F-B
  • Испанский A
  • китайский A
  • Ни один экзамен не соответствует требованиям, см. «Альтернативные способы выполнить требование» ниже
  • 2.0 по датской шкале оценок (минимальный проходной балл по отдельному предмету)
  • Документы, необходимые для оценки вышеуказанного
  • Гимназиуми еретцеги бизонитваны И
  • Gimnaziumi bizonyitvany

Расчет среднего балла:
Средний балл по всем оценкам экзаменационного сертификата ( gimnazium erettsegi bizonyitvany / Matura ) вычисляется с округлением до одного десятичного знака.

Вычисляется средний балл по всем оценкам за последний год, данный по gimnaziumi bizonyitvany , с округлением до одного десятичного знака.

Среднее значение этих двух средних баллов затем вычисляется и конвертируется в датскую шкалу оценок с использованием приведенной здесь таблицы преобразования (Ungarn; -> Karakterer. Доступно только на датском языке).

Другие экзамены

  • Szakközepiskola erettsegi bizonyitvany
  • Szakközepiskola erettsegi-kepesito bizonyitvany
  • Technikus Erettsegi-Kepesito Bizonyitvany

Если у вас есть вышеперечисленное, вы будете оценены по Квоте 2: экзамены без конвертируемого среднего балла.Вы всегда должны выполнять определенные вступительные требования. В соответствующих случаях включите обзор часов преподавания по конкретному предмету и учебной программе.

Отбор квалифицированных кандидатов в группу основан на общей оценке, при которой мы смотрим на ваши оценки в целом и с учетом конкретных требований для поступления, мы смотрим на ваши внеклассные занятия и смотрим на ваше мотивационное эссе. Мы не рассчитываем средний балл, и вы не будете оцениваться по квоте 1.

Следующая таблица сравнения действительна только для 2020 года приема.

Проверьте конкретные требования к поступлению для интересующих вас программ (в разделе выше) и сравните их с информацией в таблице ниже, чтобы узнать, выполняете ли вы.

ТРЕБОВАНИЕ

ЭКВИВАЛЕНТ

Выпускной экзамен второй ступени

Stúdentpróf

(Только для датских программ erhvervsøkonomi (HA))

Реформа 2008 г .:
мин.20 фейн (Mindst 10 Fein på rep 3 eller højere)

Реформа 1999:
Мин. 18 Эйнингар в Данске

Реформа 2008 г .:
мин. 25 фейн (Mindst 15 Fein på rep 3 eller højere)

Реформа 1999:
Мин. 18 Эйнингар

  • ENS 103 + 203 + 303 + 403 + 503 + 603

* Если у вас нет указанной выше комбинации, загрузите описание курса.

Реформа 2008 г .:
мин.15 фейн (Mindst 10 Fein på rep 3 eller højere)

Реформа 1999:
Мин. 15 Эйнингар

* Если у вас нет указанной выше комбинации, загрузите описание курса.

Реформа 2008 г .:
мин. 30 фейн (Mindst 15 Fein på rep 3 eller højere)

Reform 1999:
15-21 einingar в зависимости от комбинации:

  • СТÆ 103-203-303-403-523
  • СТÆ 103-203-303-403-513
  • СТÆ 103-203-303-313-413-363-463
  • СТÆ 103-263-313-413-363-463
  • СТÆ 103-203-303-403-503

* Если у вас нет ни одной из вышеперечисленных комбинаций, загрузите описание курса.

Реформа 2008 г .:
мин. 20 фейн (Mindst 5 Fein på rep 3 eller højere)

Reform 1999:
9-15 einingar в зависимости от комбинации:

  • СТÆ 103-203-303-403
  • СТÆ 103-203-303-313-403
  • СТÆ 103‐263‐363‐463
  • СТÆ102‐203‐363-463
  • СТÆ 103-203-303-363-463
  • СТÆ 103-203-303-313-413
  • СТÆ 103-263-313-413
  • СТÆ 103-203-303-363
  • СТÆ 103‐263‐363
  • СТÆ 103-203-363

* Если у вас нет ни одной из вышеперечисленных комбинаций, загрузите описание курса.

Реформа 2008:

  • История мин. 12 фейн (Mindst 10 Fein på rep 2 eller højere)
  • Социальные науки мин. 15 фейн (Mindst 10 Fein på rep 2 eller højere)


Реформа 1999:

  • Мин. 6 Эйнингар по истории и / или обществознанию
( Только для бакалавров в области бизнеса, языка и культуры и международного бизнеса в Азии)
  • Французский BA / F-B
  • Немецкий BA / F-B
  • Испанский A
  • китайский A

Реформа 2008:
Начальный уровень A: мин.25 фейн (Mindst 10 Fein på rep 2 eller højere)


Реформа 1999:

  • Мин. 15 эйнингар на соответствующем языке
  • 2,0 ​​по датской шкале оценок (минимальный проходной балл по одному предмету)
  • 5 (базовый модуль)
  • 4 (базовый класс)
  • C (если шкала оценок от E до A)
  • Документы, необходимые для оценки вышеуказанного
  • Stúdentpróf (диплом должен включать все коды курсов)

Расчет среднего балла:
Средний балл, указанный на Stúdentpróf , конвертируется в датскую шкалу оценок (см. Ниже).

Если на курсах Stúdentpróf, не выставлен средний балл, рассчитывается простой средний балл по всем оценкам сертификата (за исключением курсов с переводом зачетных единиц, курсов «сдал / не прошел» без оценки и Skólasókn).

Исландский средний балл преобразуется в датскую шкалу оценок с помощью приведенной здесь таблицы преобразования (Остров; -> Карактерер. Доступно только на датском языке).

Другие экзамены

  • Студенты старфснамбраутум
  • Frumgreinadeild
  • Raungreinadeilddarpróf
  • Háskolebrú

Если у вас есть вышеперечисленное, вы будете оценены по Квоте 2: экзамены без конвертируемого среднего балла.Вы всегда должны выполнять определенные вступительные требования. В соответствующих случаях включите обзор часов преподавания по конкретному предмету и учебной программе.

Отбор квалифицированных кандидатов в группу основан на общей оценке, при которой мы смотрим на ваши оценки в целом и с учетом конкретных требований для поступления, мы смотрим на ваши внеклассные занятия и смотрим на ваше мотивационное эссе. Мы не рассчитываем средний балл, и вы не будете оцениваться по квоте 1.

Следующая таблица сравнения действительна только для 2020 года приема.

Проверьте конкретные требования к поступлению для интересующих вас программ (в разделе выше) и сравните их с информацией в таблице ниже, чтобы узнать, выполняете ли вы.

ТРЕБОВАНИЕ

ЭКВИВАЛЕНТ

Выпускной экзамен второй ступени

Международный бакалавриат — диплом IB

(Только для датских программ erhvervsøkonomi (HA))
  • датский A1 (HL)
  • Датский A1 (SL)
  • Датский B (HL) мин.4 класс
  • Датская литература (HL)
  • Датская литература A (SL) мин. 4 класс
  • Норвежский A1 (HL)
  • Норвежский A1 (SL)
  • Норвежский B (HL) мин. 4 класс
  • Норвежская литература (HL)
  • Норвежская литература A (SL) мин. 4 класс
  • Шведский A1 (HL)
  • Шведский A1 (SL)
  • Шведский B (HL) мин. 4 класс
  • Шведская литература (HL)
  • Шведская литература A, (SL) мин. 4 класс
  • Фарерский A (HL)
  • Фарерский A (SL)
  • Английский A1 (HL)
  • Английский A2 (HL)
  • Английский B (HL)
  • Английский A1 (SL)
  • Английский A2 (SL)
  • English A Lang.& Лит. (HL)
  • English A Lang. & Лит. (SL)
  • Английская литература (HL)
  • Английская литература (SL)
  • Английский A1 (HL)
  • Английский A2 (HL)
  • Английский B (HL)
  • Английский A1 (SL)
  • Английский A2 (SL)
  • Английский B (SL)
  • English A Lang. & Лит. (HL)
  • English A Lang. & Лит. (SL)
  • Английская литература (HL)
  • Английская литература (SL)
  • Математика (HL)
  • Математика (SL)
  • Математика: анализ и подходы (SL)
  • Математика: приложения и интерпретация (SL)
  • История (SL или HL)
  • Экономика (SL или HL)
  • Бизнес и менеджмент (SL или HL)
  • Глобальная политика (SL или HL)
  • Социальная и культурная антропология (HL)
( Только для бакалавров в области бизнеса, языка и культуры и международного бизнеса в Азии)
  • Французский BA / F-B
  • Немецкий BA / F-B
  • Испанский A
  • китайский A

Один из следующих курсов пройден на соответствующем языке:

  • A1 (HL)
  • A2 (HL)
  • B (HL)
  • A1 (SL)
  • A2 (SL)
  • B (SL)
  • Литература A (HL)
  • Lang.& Лит. А (HL)
  • Lang. & Лит. В (HL)
  • Литература A (SL)
  • Lang. & Лит. A (SL)
  • Lang. & Лит. B (SL)
  • 2,0 ​​по датской шкале оценок (минимальный проходной балл по одному предмету)
  • Документы, необходимые для оценки вышеуказанного
  • До 2012 г. включительно: Диплом IB
  • С 2013 года включительно: Диплом международного бакалавриата И Дипломная программа Результаты
  • Онлайн-база данных: , пожалуйста, попросите вашего координатора IB предоставить CBS («Копенгагенская бизнес-школа (000151)) доступ к вашим онлайн-результатам IB

Расчет среднего балла:
Общее количество баллов конвертируется в датскую шкалу оценок с помощью приведенной здесь таблицы преобразования (Международный бакалавриат; -> Карактерер.Доступно только на датском языке).

Требования к программе International Baccalaureate:
Программа International Baccaluareate должна соответствовать всем из следующих критериев, чтобы соответствовать общим требованиям для поступления:

  • Общее количество баллов должно быть не менее 24 (включая бонусные и минус штрафные баллы)
  • Сдано минимум 6 предметов
  • Минимум 3 и не более 4 предметов, сданных на более высокий уровень (HL)
  • Все остальные курсы должны быть пройдены на дополнительном уровне (SL)
  • Расширенное эссе должно быть отправлено
  • Курс Теория познания (TOK) должен быть завершен
  • Все мероприятия CAS (творчество, действия и услуги) должны быть завершены

Следующая таблица сравнения действительна только для 2020 года приема.

Проверьте конкретные требования к поступлению для интересующих вас программ (в разделе выше) и сравните их с информацией в таблице ниже, чтобы узнать, выполняете ли вы.

ТРЕБОВАНИЕ

ЭКВИВАЛЕНТ

Выпускной экзамен второй ступени

Международный бакалавриат — результаты курса Diploma Program (DP)

(Только для датских программ erhvervsøkonomi (HA))
  • датский A1 (HL)
  • Датский A1 (SL)
  • Датский B (HL) мин.4 класс
  • Датская литература (HL)
  • Датская литература A (SL) мин. 4 класс
  • Норвежский A1 (HL)
  • Норвежский A1 (SL)
  • Норвежский B (HL) мин. 4 класс
  • Норвежская литература (HL)
  • Норвежская литература A (SL) мин. 4 класс
  • Шведский A1 (HL)
  • Шведский A1 (SL)
  • Шведский B (HL) мин. 4 класс
  • Шведская литература (HL)
  • Шведская литература A, (SL) мин. 4 класс
  • Фарерский A (HL)
  • Фарерский A (SL)
  • Английский A1 (HL)
  • Английский A2 (HL)
  • Английский B (HL)
  • Английский A1 (SL)
  • Английский A2 (SL)
  • English A Lang.& Лит. (HL)
  • English A Lang. & Лит. (SL)
  • Английская литература (HL)
  • Английская литература (SL)
  • Английский A1 (HL)
  • Английский A2 (HL)
  • Английский B (HL)
  • Английский A1 (SL)
  • Английский A2 (SL)
  • Английский B (SL)
  • English A Lang. & Лит. (HL)
  • English A Lang. & Лит. (SL)
  • Английская литература (HL)
  • Английская литература (SL)
  • Математика (HL)
  • Математика (SL)
  • Математика: анализ и подходы (SL)
  • Математика: приложения и интерпретация (SL)
  • История (SL или HL)
  • Экономика (SL или HL)
  • Бизнес и менеджмент (SL или HL)
  • Глобальная политика (SL или HL)
  • Социальная и культурная антропология (HL)
( Только для бакалавров в области бизнеса, языка и культуры и международного бизнеса в Азии)
  • Французский BA / F-B
  • Немецкий BA / F-B
  • Испанский A
  • китайский A

Один из следующих курсов пройден на соответствующем языке:

  • A1 (HL)
  • A2 (HL)
  • B (HL)
  • A1 (SL)
  • A2 (SL)

Среднее образование | Британника

Среднее образование , вторая ступень, традиционно относящаяся к формальному образованию, которая начинается примерно в возрасте от 11 до 13 лет и заканчивается обычно в возрасте от 15 до 18 лет.Дихотомия между начальным и средним образованием постепенно стала менее заметной не только в учебных программах, но и в организации. Распространение средних школ, неполных школ, неполных средних школ и других подразделений привело к появлению систем с более чем двумя ступенями.

Подробнее по этой теме

Образование: Среднее

В европейских системах образования среднее образование было в первую очередь подготовкой к университету, преследуя общие цели и идеалы…

Из-за их широкого влияния во всем мире уместно вкратце обрисовать образовательные модели нескольких крупных европейских стран и Соединенных Штатов.

Франция инициировала ряд образовательных реформ, начавшихся в 1968 году. Современная Франция имеет первый цикл образования для всех детей до 5 класса. За переходными 6 и 7 классами следуют 8 и 9 классы, которые составляют так называемый цикл ориентации. , в котором студенты изучают определенную базовую программу, второй иностранный язык и набор факультативов.К концу 9 класса они должны решить, продолжать ли учебу в старших классах средней школы или выбрать профессиональное образование. В принципе, родители, ученики и школьные консультанты должны прийти к соглашению по поводу этого решения, но существуют процедуры обжалования для разрешения разногласий.

Учащиеся в возрасте от 15 до 18 лет поступают в два лицея или среднюю школу: (1) лицей общего и технологического образования ( lycée d’enseignement général et technologique, или LEGT) является преемником традиционного академического лицея. прошлого; (2) профессионально-образовательный лицей ( lycée d’enseignement Professionalnel, или LEP) охватывает ряд профессионально-технических исследований и обучения.Учебная программа LEGT начинается в 10 классе с определенных отслеживаемых базовых курсов, которые ведут в 11 и 12 классах к специализации в любой из пяти предметных областей: литературно-философские исследования, экономика и социальные науки, математика и физические науки, науки о Земле и биологические науки. науки, а также научно-производственные технологии. В 10 и 11 классах есть общее ядро ​​предметов плюс варианты, но в 12 классе все предметы, будучи необязательными, ориентированы на основную область обучения ученика.Экзамен на степень бакалавра и , сданный в конце этих исследований, позволяет студентам поступать в университет.

Получите эксклюзивный доступ к контенту из нашего первого издания 1768 с вашей подпиской. Подпишитесь сегодня

Профессионально-техническое среднее образование включает трехлетний выбор факультативных курсов, ведущих к получению одного из 30 или около того технических бакалавров. Студент может вместо этого выбрать годичный курс, не дающий особой квалификации, или может выбрать ученичество на рабочем месте.

В Германии школы находятся в ведении правительств штатов, или Länder, , и структура начального и среднего образования не совсем едина во всей стране. Однако во всех штатах период начального образования охватывает восемь или девять классов. По истечении этого срока ученикам доступны три основных варианта. После консультации с учителем начальной школы и по просьбе родителей они могут быть помещены в школу Real , гимназию , или Hauptschule, последнюю из которых представляют для продолжения начального образования.

Учащиеся Hauptschule продолжают изучение языка, арифметики, географии, истории, естественных наук, музыки, искусства и физического воспитания. После завершения четырех- или пятилетней программы обучения в Hauptschule, ученик, как правило, переходит на профессиональную подготовку.

В Германии термин «средняя школа» относится к учебным заведениям, предлагающим курсы, ведущие к «Сертификату зрелости» ( Reifezeugnis ), квалификации для поступления в высшее учебное заведение. Realschule предлагает ученикам дополнительное общее образование, некоторые курсы повышения квалификации и изучение английского языка. В возрасте 16 лет учащиеся завершают программу обучения и переходят в профессионально-техническое училище или поступают на производственное обучение.

Ученик, имеющий академическую квалификацию, может также перейти в гимназию . Гимназия , третья альтернатива для немецкой молодежи, предлагает серьезную академическую подготовку к высшему образованию. Как и лицей во Франции и гимназия в Англии, Gymnasium предназначена для тех учащихся, которые продемонстрировали самые академические успехи; и его учебная программа, в которой особое внимание уделяется языкам, математике, естественным и общественным наукам, требует высокой степени усердия во всех девяти классах.Неуспевающие ученики гимназии могут быть переведены в Hauptschule. Кроме того, по достижении 16-летнего возраста учащиеся могут прекратить учебу и поступить в профессионально-техническое училище.

Учащиеся гимназии должны сдать экзамен Abitur, , дающий им право на получение аттестата зрелости, если они хотят поступить в немецкий университет. Содержание Abitur адаптировано к направленности обучения, например, классическим языкам или математике, выбранным ранее студентом.

Нынешняя система среднего образования Великобритании является преемницей немного более старой системы, в которой учащиеся в возрасте 11 лет распределялись в один из трех типов школ посредством отборочных тестов. С 1950-х и 60-х годов эти школы были постепенно заменены школами единого типа, называемыми общеобразовательными школами, которые принимают детей независимо от способностей и способностей и которые предлагают как академические, так и профессиональные программы под одной крышей.

В Великобритании первая ступень образования называется начальным образованием и включает учащихся в возрасте от 5 (4 в Северной Ирландии) до 11 (12 в Шотландии).Около 90 процентов учащихся затем посещают общеобразовательные школы. Эти школы организованы по-разному и предназначены для детей от 11 до 18 лет; 11–12–16; или с 12–14 до 16–18. Большинство оставшихся учащихся получают среднее образование в современных средних школах или гимназиях (они являются остатками старой трехсторонней школьной системы), куда они направляются после выборочных процедур в возрасте 11 лет.

Закон о реформе образования 1988 г. установил национальную учебную программу и регулярную оценку успеваемости детей в возрасте от 5 до 16 лет во всех государственных школах Англии и Уэльса.Аналогичное законодательство было принято в Северной Ирландии в 1990 году. В Англии, Шотландии и Уэльсе родители имеют законное право выражать предпочтение определенной школе.

Студенты, желающие поступить в университет, должны успешно сдать серию экзаменов, по результатам которых выдается Общий аттестат об образовании. Эти экзамены бывают двух уровней: аттестат об общем среднем образовании (GCSE; ранее «обычный») и продвинутый. Для поступления в университет требуется обязательная комбинация сдачи экзамена GCSE и продвинутого уровня по таким предметам, как английский, иностранный язык, естественные науки и математика.Учебная программа шестого класса (т. Е. Двух последних лет средней школы) в основном ориентирована на подготовку к экзамену продвинутого уровня и предусматривает углубленную специализацию.

В Великобритании также существует небольшая, обособленная, но очень значительная группа независимых школ. Эти начальные и средние школы являются самоокупаемыми. Самыми известными из независимых школ являются «государственные школы», некоторые из которых, особенно Итон и Харроу, уже давно пользуются выдающейся репутацией.Эти школы на протяжении веков готовили студентов академически к получению высшего образования, обычно в университетах Оксфорда и Кембриджа, и, в конечном итоге, к лидерству в британской жизни. Хотя эти учебные заведения являются спорным элементом в британском образовании и часто обвиняются в усилении оскорбительных социальных различий, они остаются популярными.

Современные нации, когда-то составлявшие СССР, имеют образовательный образец, который сильно отличается от только что описанного. Первый этап обучения проходит в восьмилетней общеобразовательной школе.После окончания восьмилетней школы учащиеся, если они имеют соответствующую квалификацию, имеют ряд открытых для них средних школ, включая политехнические или общеобразовательные школы, а также различные профессиональные или технические школы. Органы образования пытаются обеспечить продолжительное руководство для учеников и направить их на все более дифференцированные программы обучения на протяжении последних нескольких лет начальной школы и первых двух или трех лет средней школы. Многие из тех, кто бросает начальную школу, сразу же приступают к работе.Наиболее важным для студентов, желающих продолжить свое образование, является отбор в 17 или 18 лет для получения высшего образования. Университеты и высшие учебные заведения принимают от 15 до 20 процентов выпускников средних школ. Большинство выпускников политехнических или общеобразовательных средних школ должны устроиться на работу в течение двух или трех лет, после чего получившие квалификацию могут перейти к высшему образованию.

Система США

Исторически сложилось так, что в Соединенных Штатах мало различий между начальным и начальным образованием, т.е.е., между теми низшими школами, которые готовили учащихся к продвинутому (или высшему) образованию, и теми школами, которые просто обеспечивали грамотность и некоторое общее образование для больших групп детей. Это наследие привело в Соединенных Штатах к единой системе начального и среднего образования для подавляющего большинства студентов.

В Соединенных Штатах ответственность за образование лежит на отдельных штатах; поэтому существуют некоторые вариации в структуре. Однако обычно учащиеся в возрасте 13 или 14 лет поступают в среднюю школу по соседству; обучение там обычно длится четыре года, и студенты заканчивают обучение в среднем в 18 лет.

В Соединенных Штатах более 80 процентов групп среднего возраста посещают среднюю школу. Американская цель на протяжении десятилетий заключалась в том, чтобы предоставить среднее образование для всех в рамках единого типа учебного заведения, обычно называемого общеобразовательной средней школой. Хотя точное значение этого термина часто неясно, общеобразовательные школы обычно включают широкую программу общих и специализированных учебных программ и учитывают широкий диапазон академических способностей учеников.Таким образом, в той же американской средней школе некоторые студенты планируют продолжить обучение в университете, а другие хотят получить окончательное среднее образование.

Эта статья была последней отредактированной и обновленной Майклом Рэем, редактором.

Узнайте больше в этих связанных статьях Britannica:

  • Образование: Среднее

    В европейских системах образования среднее образование было преимущественно подготовкой к университету, с целями и идеалами общей культуры, которые радикально и с самого начала отличали его от образования начального типа.Вплоть до начала 20-го…

  • Образование: Среднее

    Между начальной школой и различными типами высшего образования эллинистическая система образования ввела программу промежуточного подготовительного обучения — предварительное образование, своего рода общий канал подготовки к различным отраслям высшей культуры, enkyklios paideia («общее, или общее, образование »).

Leave a Reply

Добавить комментарий

Ваш адрес email не будет опубликован. Обязательные поля помечены *